"
Toán tuổi thơ 2 THCS Số 185 và 186 phát hành tháng 7,8 năm 2018
🔙 Quay lại trang tải sách pdf ebook Toán tuổi thơ 2 THCS Số 185 và 186 phát hành tháng 7,8 năm 2018
Ebooks
Nhóm Zalo
HUAN CHU
VIỆT NAM
LAO DONG
NĂM THỨ
MƯỜI CHÍN
ISSN 1859-2740
Trang
tuổi thời
185
9
186
+
NĂM HỌC 2018 - 2019
TRUNG HỌC CƠ SỞ
NHÀ XUẤT BẢN GIÁO DỤC VIỆT NAM - BỘ GIÁO DỤC VÀ ĐÀO TẠO
GD
A
LE BÉ MAC
HONG HA
CUỘC THI CÂU LẠC BỘ TOÀN TUỔI THU TOÀN QUỐC 2018
CHÚC THẦY CÔ VÀ CÁC BẠN HỌC SINH MỘT NĂM HỌC THÀNH CÔNG !
e
HỘI ĐỒNG BIÊN TẬP MỚI CỦA TẠP CHÍ TOÁN TUỔI THƠ 28
(Theo Quyết định số 520/QĐ-NXBGDVN ngày 05/9/2018 của Chủ tịch Hội đồng thành viên Nhà xuất bản Giáo dục Việt Nam)
TS. Trần Quang Vinh
ThS. Nguyễn Ngọc Hân
Nhà giáo Trần Thị Kim Cương
NGND. Vũ Hữu Bình
TS. Nguyễn Minh Đức
ThS. Đặng Hiệp Giang
TS. Nguyễn Minh Hà
PTS. TS. Vũ Đình Hòa
ThS. Trần Quang Hùng
TS. Lê Thống Nhất
PGS. TS. Tạ Duy Phượng
ThS. Phạm Đức Tài
NGND. PGS. TS. Tôn Thân
PGS. TS. Lê Anh Vinh
Toán
tuổi thơ 2
TRUNG HỌC CƠ SỞ
Children's Fun Maths Journal
NHÀ XUẤT BẢN GIÁO DỤC VIỆT NAM - BỘ GIÁO DỤC VÀ ĐÀO TẠO
HỘI ĐỒNG BIÊN TẬP
Phó Tổng biên tập NXBGD Việt Nam:
TS. TRẦN QUANG VINH
Phó Tổng biên tập phụ trách tạp chí: ThS. NGUYỄN NGỌC HÂN Phó Tổng biên tập tạp chí: TRẦN THỊ KIM CƯƠNG
ỦY VIÊN
NGND. VŨ HỮU BÌNH TS. NGUYỄN MINH ĐỨC
ThS. ĐẶNG HIỆP GIANG TS. NGUYỄN MINH HÀ PGS. TS. VŨ ĐÌNH HÒA ThS. TRẦN QUANG HÙNG
TS. LÊ THỐNG NHẤT
PGS. TS. TẠ DUY PHƯỢNG
ThS. PHẠM ĐỨC TÀI NGND. PGS. TS. TÔN THÂN
PGS. TS. LÊ ANH VINH
CHỊU TRÁCH NHIỆM XUẤT BẢN
Chủ tịch Hội đồng Thành viên NXBGD Việt Nam: NGUYỄN ĐỨC THÁI
Tổng Giám đốc NXBGD Việt Nam: HOÀNG LÊ BÁCH
Phó Tổng Giám đốc kiêm Tổng biên tập NXBGD Việt Nam: PHAN XUÂN THÀNH
TRONG SỐ NÀY
Bài phát biểu tại Lễ khai mạc của Trưởng ban tổ chức cuộc thi Câu lạc bộ Toán Tuổi thơ toàn quốc 2018
Tr 8
Bài phát biểu tại Lễ bế mạc của Trưởng ban tổ chức cuộc thi Câu lạc bộ Toán Tuổi thơ toàn quốc 2018
Thư cảm ơn
Tr 10
Tr 11
Cuộc thi Câu lạc bộ Toán Tuổi thơ toàn quốc 2018
Đề thi cá nhân bậc THCS
Tr 12
Đáp án đề thi cá nhân bậc THCS
Tr 14
Đề thi vòng 1 (Tiếp sức Toán)
Tr 15
Đề thi vòng 2 (Du lịch Toán học)
Tr 15
Kết quả cuộc thi Câu lạc bộ Toán Tuổi thơ
TÒA SOẠN
toàn quốc 2018 - THCS
Tr 17
Lời giải chi tiết đề thi cá nhân THCS
Tr 20
Tầng 2, nhà A, số 187B Giảng Võ, phường Cát Linh, quận Đống Đa, Hà Nội
Giải toán thế nào?
Tr 24
Điện thoại: 024.35682701 - Fax: 024.35682702 Email (Ban biên tập): bbttoantuoitho@gmail.com Email (Trị sự - Phát hành): tapchitoantuoitho@gmail.com Website: http://www.toantuoitho.vn
ĐỐI TÁC ĐẠI DIỆN PHÍA NAM
Công ty cổ phần Đầu tư và Phát triển Giáo dục Phương Nam 231 Nguyễn Văn Cừ, Q.5, TP. Hồ Chí Minh ĐT: 028.38357197, Email: thitruong@phuongnam.edu.vn
Trị sự - Phát hành
TRỊNH THỊ TUYẾT TRANG,
NGUYỄN THỊ HUYỀN THANH, NGUYỄN THỊ HẢI ANH
Biên tập - Chế bản: VŨ THỊ MAI, ĐỖ TRUNG KIÊN Mĩ thuật: TRẦN NGỌC TRƯỜNG
Sử dụng tính chất đường trung tuyến ứng với
cạnh huyền của tam giác vuông
Thái Nhật Phượng
Compa vui tính Vẽ thế nào nhỉ? Nguyễn Xuân Bình
Đo trí thông minh Điền số thích hợp Nguyễn Đức Tấn
Tr 26
Tr 27
Đề thi tuyển sinh lớp 10 TP. Hà Nội năm học
2018-2019
Tr 28
Nhìn ra thế giới
Tr 30
Một góc nhìn hai bài hình trong đề thi IMO 2018 Nguyễn Bá Đang
Vào thăm Vườn Anh
Ô chữ Birds
Hoàng Thị Phượng
Kết quả Thi giải toán qua thư
Tr 32
Tr 33
Kết quả kì thi Olympic quốc tế của học
sinh Việt Nam năm 2018
Tr 36
Lịch sử Toán học
Tr 37
Toán chuyển động đều trong hai cuốn sách
toán cổ Hán Nôm nước ta
Tạ Duy Phượng, Đoàn Thị Lệ, Cung Thị Kim Thành, Phan Thị Ánh Tuyết
Vượt vũ môn
Bất đẳng thức và cực trị đại số qua kì thi tuyển sinh vào lớp 10 chuyên Toán năm học
2018-2019
Chữ và chữ số
Tr 49
Kì 34
Nguyễn Đức Tấn
Tr 39
Vài phương pháp giải phương trình vô tỉ
Đông Ba
Phạm Minh Tú
Tr 42
Phá án cùng thám tử Sê Lốc Cốc
Tr 50
Nhìn ra thế giới
Vụ án trước cửa hàng sách
Tr 44
Bài toán hay về phần nguyên của một số
Lê Hồng Mai
Giải toán thế nào?
Tr 52
Trương Quang An
Sai ở đâu? Sửa cho đúng
Một số dạng toán về tỉ lệ thức
Tr 47
Hà Văn Nhân
Lời giải đúng chưa?
Thách đấu
Tr 55
Đinh Văn Thư
Trận đấu thứ một trăm năm mươi lăm
Toán
tuoi the.
Võ Quốc Bá Cẩn
Dành cho các nhà toán học nhỏ
Tr 56
2
Mở rộng và khai thác một số bài toán tổ hợp
Trịnh Hoài Dương, Lê Đình Trường
Giải toán học Anh
Perpendicular lines and parallel lines
Tr 61
Trịnh Hoài Dương, Nguyễn Thành Nam,
Hoàng Anh Quân
Thì thầm... Thì thầm thôi...
Tr 62
Anh Phó Gỡ xưa
Thi giải toán qua thư
Tr 63
Ảnh bìa 1: Các vị đại biểu chụp ảnh với các
thí sinh đoạt Huy chương Vàng cuộc thi Câu
lạc bộ Toán Tuổi thơ toàn quốc 2018
Phóng sự ảnh
THỂ LO HOẠT MẠC
CUỘC THI CÂU LẠC BỘ TOÁN TUỔI THƠ TOÀN QUỐC 20
LEKHAT MAC
THE TOAN QING 2018
LEKHAI MAC
100 2018
2018
TS. Phan Xuân Thành, Phó Tổng Giám đốc kiêm Ông Lê Quang Minh, Chủ tịch Ủy ban nhân dân
Tổng biên tập NXBGD Việt Nam
thành phố Lào Cai
LEKHAI MAC
卷
LE KHAI MAC
OLTO TOAN QUOG 2018
Ông Nguyễn Ngọc Hân, Trưởng Ban tổ chức cuộc thi Câu lạc bộ Toán Tuổi thơ toàn quốc 2018
Ông Hoàng Mạnh Ánh, Phó Tổng Giám đốc Công ty cổ phần Văn phòng phẩm Hồng Hà
LỄ KHAI MẠC
GIOI THI SAU THÌ TO TOÀN THÌ Outlo
Tiết mục văn nghệ chào mừng các đoàn về dự thi
3
Các đại biểu dự Lễ khai mạc
THỄ KHAI MẠC CUỘC THI CÂU LẠC BỘ TOÁN TUỔI THƠ TOÀN QUỐC 2018
Các đoàn diễu hành qua sân khấu
LE KHAI MAC
CUỘC THI CÂU THỂ BỎ TOÀN TUỔI TRỞ TRÊN QUuốt min
Ban tổ chức trao Kỉ niệm chương cho đơn vị đăng cai
LỄ KHAI MẠ
CUỘC THI ĐẦU LẠC BỘ TOÀN VỚI THE this butt clo
LE KHAI NAC
Đại diện nhà tài trợ chính nhận Kỉ niệm chương của Ban tổ chức
Đơn vị đăng cai tặng quà cho các đoàn về tham dự cuộc thi
19
LU KHAI MAC
CT THE ƯU TIÊN TU THU DVB T
百
LE KHAI MAC
CUỘC THI CÂU ĐỐI DO TUẦN TUỔI THỊ TOÀN QUỐC IN H
四
Các đoàn tham dự nhận cờ lưu niệm
4
Ban tổ chức trao thưởng cuộc thi Giải toán qua thư năm học 2017 - 2018
PHẦN THI CÁ NHÂN VÀ TIẾP SỨC TOÁN
Các thí sinh tại trường thi - trường tiểu học Bắc Cường
Các vị đại biểu, Trưởng đoàn, Lãnh đội,
Giám khảo, Giám thị
Các thí sinh háo hức và hồi hộp chờ đến phần thi cá nhân
Cảm xúc vui buồn sau phần thi cá nhân
Các thí sinh đang làm bài thi cá nhân
LO
5
Phần thi Tiếp sức Toán đầy căng thẳng
OI B
Thi Tiếp sức Toán
PHẦN THỰC DU LỊCH TOAN THUỘC
Các đội trưởng bốc thăm xem đội mình mang tên nhà toán học nào
PHỦ THỦ
Đội trưởng nộp kết quả tại thành phố nhận đề
HANOI A
Cả đội cùng trao đổi để tìm kết quả của bài toán
6
LỄ BẾ MẠC CUỘC THI CÂU LẠC BỘ TẤT TOÀN QUỐC 2018
ĐANG GIỐNG GIAN VIỆT NAM GIANG VINH NGOIN
Văn nghệ mừng thành công cuộc thi Câu lạc bộ Toán Tuổi thơ toàn quốc 2018
348
LE BE MAC
Trao giải phần thi cá nhân
LE BE MAC
CUOD THE GAU LAC BO TOAN TUOI THE TUAN GUDC 2018.
LE BE MAC
CUOI THI BẦU LAU BU THẦN THÁI THỊ TOÀN QUỐC PHI
Trao giải phần thi Tiếp sức Toán và Du lịch Toán học
368
LE BE MAC
HUDE THI THU LẠI DO TOÁN TUỔI TED TOÀN BUTE TUIS
Trao giải cho ba thí sinh
giải đúng liên tiếp nhiều câu nhất tính từ câu 1
7
LE BE MAC
HUỚC THÌ SAU LẠC BỘ TUẦN TUỔI THÌ THẦM DÙNG 2018
Trao giải cho các thí sinh
đạt điểm cao nhất
BÀI PHÁT BIỂU TẠI LỄ KHAI MẠC CỦA TRƯỞNG BAN TỔ CHỨC
CUỘC THI CÂU LẠC BỘ TOÁN TUỔI THƠ TOÀN QUỐC 2018
Kính thưa các vị đại biểu, các vị khách quý, các vị trưởng đoàn, lãnh đội, các em học sinh, các bậc phụ huynh và các phóng viên báo, đài.
Tờ Tạp chí Toán Tuổi thơ đầu tiên ra mắt độc giả năm 2000. Khi đó, mới chỉ là đặc san của Tạp chí Toán học và Tuổi trẻ. Năm 2002, tạp chí Toán Tuổi thơ chính thức được thành lập. Ngay từ khi mới ra đời, Tạp chí Toán Tuổi thơ đã nhanh chóng được các thầy cô giáo, các em học sinh ở tiểu học và trung học cơ sở yêu mến. Từ năm 2005 đến năm 2014, Tạp chí đã phối hợp với các Sở Giáo dục và Đào tạo tổ chức được 3 lần Giao lưu Toán Tuổi thơ tại Nam Định, Quảng Ninh, Hải Phòng, 7 lần Olympic Toán Tuổi thơ toàn quốc tại Hà Nội, Thừa Thiên - Huế, Long An, Lào Cai, Cà Mau, Vĩnh Phúc và Đắk Lắk với 53 tỉnh, thành tham dự. Rất nhiều tỉnh, thành đã tổ chức các kì thi toán cấp huyện, cấp tỉnh theo mô hình tổ chức của Giao lưu Toán Tuổi thơ và Olympic Toán Tuổi thơ. Chúng tôi tự hào vì đã tạo ra một sân chơi trí tuệ, lành mạnh, bổ ích cho các em học sinh và các thầy cô giáo.
Từ năm học 2015-2016, tạp chí phối hợp với các Sở Giáo dục và Đào tạo tổ chức Câu lạc bộ Toán Tuổi thơ trong các nhà trường để tạo phong trào dạy và học toán bằng tiếng Anh. Nhiều địa phương đã tổ chức Cuộc thi Câu lạc bộ Toán Tuổi thơ cấp huyện và cấp tỉnh. Tháng 6.2016, Tạp chí đã tổ chức thành công cuộc thi Câu lạc bộ Toán Tuổi thơ toàn quốc tại thủ đô Hà Nội với 24 tỉnh thành trong cả nước tham dự. Tháng 6.2017, Cuộc thi Câu lạc bộ Toán Tuổi thơ toàn quốc được tổ chức tại tỉnh Trà Vinh với sự góp mặt của 24 tỉnh thành đến từ ba miền Bắc, Trung, Nam.
Tiếp nối thành công đó, năm nay Cuộc thi Câu lạc bộ Toán Tuổi thơ toàn quốc trở lại tổ chức tại thành phố Lào Cai xinh đẹp và mến khách. Lào Cai là tỉnh có ngành giáo dục phát triển mạnh, đặc biệt rất thành công trong việc tổ chức các Câu lạc bộ toán. Năm nay có 294 thí sinh từ 20 tỉnh thành trong cả nước cùng tham dự. Đó là các tỉnh: Bắc Giang, Đà Nẵng, Hà Nam, Hà Nội, Hải Dương, Hòa Bình,
8
Hưng Yên, Kiên Giang, Lạng Sơn, Lào Cai, Nam Định, Ninh Bình, Phú Thọ, TP. Hồ Chí Minh, Sơn La, Thái Bình, Thanh Hóa, Trà Vinh, Tuyên Quang và Vĩnh Phúc. Tính đến nay đã có 60/63 tỉnh, thành tham dự các cuộc thi toàn quốc do Toán Tuổi thơ tổ chức, điều đó cho thấy sức hấp dẫn và uy tín của cuộc thi. Cuộc thi năm nay cả ở Tiểu học và Trung học cơ sở đều diễn ra các phần thi: Cá nhân, Tiếp sức Toán và Du lịch Toán học. Đề thi ở tất cả các phần thi là toán bằng tiếng Anh, các thí sinh không sử dụng máy tính bỏ túi khi làm bài.
- Ở phần thi cá nhân: Mỗi thí sinh cần thể hiện được sự nhanh nhẹn và chính xác khi làm bài thi gồm 16 câu trong thời gian 30 phút, trong đó 15 câu đầu chỉ ghi đáp số, câu 16 trình bày lời giải bằng tiếng Anh.
- Trong phần thi Tiếp sức Toán: Các em học sinh trong các đội được tham gia lần lượt giải 6 bài toán trong thời gian không quá 30 phút giống như thi chạy tiếp sức.
- Mỗi cấp học có 8 đội xuất sắc nhất trong phần thi Tiếp sức Toán được tham gia phần thi Du lịch Toán học. Ở phần thi này cả đội cần thể hiện khả năng hợp tác nhóm để cùng giải từng bài toán cho đến khi tìm ra kết quả đúng của bài đó thì mới có tấm vé đến du lịch ở thành phố tiếp theo. Sẽ thật tuyệt vời nếu cả đội được đi du lịch ở cả 6 thành phố.
Hi vọng cùng với việc được gặp gỡ, học hỏi qua cách thi mới lạ, hấp dẫn, các bạn học sinh đến từ 20 tỉnh, thành trong cả nước, được khẳng định mình và học hỏi để cùng tiến bộ. Các bạn nhỏ từ miền Trung, miền Nam sẽ được tham quan, trải nghiệm đầy thú vị các danh lam thắng cảnh của miền núi phía Bắc. Cuộc thi chắc chắn sẽ là kỉ niệm đẹp với tất cả mọi người đặc biệt là các em học sinh. Đó chính là tâm huyết, là thành công chung của tất cả chúng ta.
Thay mặt ban lãnh đạo tạp chí Toán Tuổi thơ, tôi xin cảm ơn lãnh đạo Bộ Giáo dục và Đào tạo, NXBGD Việt Nam, UBND tỉnh Lào Cai, UBND TP. Lào Cai, Sở GD - ĐT Lào Cai và Phòng GD - ĐT thành phố Lào Cai đã ủng hộ giúp đỡ cho Cuộc thi Câu lạc bộ Toán Tuổi thơ toàn quốc 2018; xin cảm ơn Công ty cổ phần Văn phòng phẩm Hồng Hà - nhà tài trợ chính đã liên tục tài trợ cho Câu lạc bộ Toán Tuổi thơ toàn quốc trong nhiều năm qua; cảm ơn các nhà tài trợ; cảm ơn các cơ quan thông tấn báo chí đã đến dự và đưa tin...
Xin chúc sức khỏe các vị đại biểu, chúc các bạn học sinh đạt được thành tích cao.
9
BÀI PHÁT BIỂU TẠI LỄ BẾ MẠC CỦA TRƯỞNG BAN TỔ CHỨC CUỘC THI CÂU LẠC BỘ TOÁN TUỔI THƠ TOÀN QUỐC 2018
Kính thưa các vị đại biểu, các vị khách quý, các vị trưởng đoàn, lãnh đội, các em học sinh, các bậc phụ huynh và các phóng viên báo, đài.
Những ngày qua, tại thành phố Lào Cai xinh đẹp và mến khách, 294 em học sinh đến từ 20 tỉnh, thành trong cả nước đã tụ hội về đây để tham dự một hoạt động thường niên, Cuộc thi Câu lạc bộ Toán Tuổi thơ toàn quốc mà các em đã mong chờ. Tham gia cuộc thi, các em học sinh được khẳng định mình, được giao lưu, học hỏi và tham gia các phần thi mới lạ, đề thi toán bằng tiếng Anh tiệm cận với đề thi của các nước và khu vực, giúp các em dần làm quen và từng bước tập dượt hội nhập quốc tế.
Sự thành công của cuộc thi là kết quả của công tác chuẩn bị hết sức nghiêm túc, tỉ mỉ và sự tham gia nhiệt tình, chăm lo chu đáo cho các em học sinh của các vị trưởng đoàn, lãnh đội, các vị phụ huynh. Sự nhiệt tình chu đáo và mến khách của đơn vị đăng cai, sự đồng hành của nhà tài trợ chính – Công ty cổ phần Văn phòng phẩm Hồng Hà.
Thành công lớn nhất của cuộc thi là khơi dậy phong trào dạy và học toán, đặc biệt là dạy toán bằng tiếng Anh ở các Câu lạc bộ toán trong các nhà trường. Thay mặt Ban tổ chức tôi xin cảm ơn Bộ Giáo dục và Đào tạo, Nhà xuất bản Giáo dục Việt Nam, UBND thành phố Lào Cai, Sở Giáo dục và Đào tạo Lào Cai, Phòng Giáo dục và Đào tạo thành phố, Ban lãnh đạo và cán bộ công nhân viên trường tiểu học Bắc Cường, cảm ơn 50 bạn tình nguyện viên đã nhiệt tình giúp đỡ Ban tổ chức. Đặc biệt xin cảm ơn Công ty cổ phần VPP Hồng Hà, nhà tài trợ chính của cuộc thi trong nhiều năm qua, cảm ơn các nhà tài trợ. Cảm ơn các phóng viên VTV, các cơ quan báo chí truyền thông đã đến dự và đưa tin. Cảm ơn các Sở Giáo dục và Đào tạo của 20 tỉnh thành đã cử đoàn tham dự, cảm ơn các trưởng đoàn, lãnh đội, các em học sinh và các bậc phụ huynh. Tất cả đã chung tay làm nên cuộc thi Câu lạc bộ Toán Tuổi thơ toàn quốc có nhiều ý nghĩa và trở thành cuộc thi uy tín được tổ chức thường niên.
Dù kết quả cuộc thi như thế nào, Ban tổ chức Cuộc thi Câu lạc bộ Toán Tuổi thơ toàn quốc 2018 tin rằng các quý vị đại biểu và các em học sinh luôn nhớ về những ngày đầu tháng 6 tại thành phố Lào Cai với phong cảnh hùng vĩ và thơ mộng này. Thời gian diễn ra cuộc thi tuy ngắn ngủi nhưng có đủ mọi cung bậc cảm xúc, hồi hộp có, hạnh phúc có, có lúc lại vui buồn đan xen. Câu lạc bộ Toán Tuổi thơ toàn quốc năm nay có nhiều nét ấn tượng: Đề thi cá nhân ở cấp Tiểu học được đánh giá là khó hơn năm trước, một số bài toán trong các đề thi được đánh giá là khá hay và đòi hỏi các thí sinh cần có kiến thức tốt và khả năng hợp tác nhóm để giải quyết vấn đề. Chúng ta được chứng kiến chiến thuật làm bài của các đội khá khác nhau trong phần thi Tiếp sức Toán. Một số đội làm bài rất nhanh để mong được cộng 1 điểm, một số đội do không kiểm tra kĩ nên đã làm sai nhiều bài. Có đội lại chia thời gian để sử dụng hết 30 phút cho 6 bài toán. Đặc biệt trong phần thi Du lịch Toán học, các thí sinh được tham gia một phần thi tuyệt vời, vừa làm bài vừa di chuyển đến các thành phố trong sân có mái che giữa lúc cơn mưa chưa tạnh. Đến với Cuộc thi, các em đã có một kỉ niệm đẹp của tuổi học trò.
Sau đây là giờ phút mong đợi nhất của các thí sinh. Xin mời MC tiếp tục chương trình.
Xin cảm ơn các vị đại biểu, các vị khách quý, các thầy cô giáo và các em học sinh. Hẹn gặp lại ở cuộc thi Câu lạc bộ Toán Tuổi thơ toàn quốc năm 2019.
10
THƯ CẢM ƠN
Cuộc thi Câu lạc bộ Toán Tuổi thơ toàn quốc 2018 đã diễn ra trong ba ngày từ 8/6/2018 đến 10/6/2018 tại thành phố Lào Cai, tỉnh Lào Cai với sự tham dự của 28 đoàn gồm 27 đội Tiểu học, 22 đội THCS đến từ 20 tỉnh thành trong cả nước.
Cảm ơn đơn vị đăng cai - UBND thành phố Lào Cai, Sở Giáo dục và Đào tạo Lào Cai, Phòng Giáo dục và Đào tạo thành phố Lào Cai, trường tiểu học Bắc Cường đã tạo điều kiện giúp đỡ để cuộc thi thành công tốt đẹp.
Cảm ơn lãnh đạo nhiều Sở Giáo dục và Đào tạo, lãnh đạo Thành ủy, UBND thành phố Lào Cai, lãnh đạo NXBGDVN và nhiều nhà tài trợ đã dự Lễ Khai mạc và Bế mạc.
Cảm ơn TS. Tạ Ngọc Trí, Phó Vụ Trưởng Vụ Giáo dục Tiểu học, Bộ Giáo dục và Đào tạo đã gửi lẵng hoa chúc mừng Cuộc thi.
Cảm ơn các đồng chí lãnh đạo, chuyên viên các phòng Giáo dục và Đào tạo có đoàn dự thi, các trưởng đoàn, lãnh đội, các thầy cô giáo, các bậc phụ huynh và 294 học sinh đại diện cho các Câu lạc bộ Toán Tuổi thơ ở các tỉnh, thành phố: Bắc Giang; Đà Nẵng; Hà Nam; Hà Nội; Hải Dương; Hòa Bình; Hưng Yên; Kiên Giang; Lạng Sơn; Lào Cai; Nam Định; Ninh Bình; Phú Thọ; TP. Hồ Chí Minh; Sơn La; Thái Bình; Thanh Hóa; Trà Vinh; Tuyên Quang; Vĩnh Phúc đã tham dự Cuộc thi.
Cảm ơn nhà tài trợ chính - Công ty Cổ phần Văn phòng phẩm Hồng Hà và các nhà tài trợ: Nhà xuất bản Giáo dục Việt Nam, Nhà xuất bản Giáo dục tại TP. Đà Nẵng; Công ty Cổ phần Đầu tư và phát triển Giáo dục tại TP. Đà Nẵng; Công ty Cổ phần Sách Giáo dục tại TP. Hà Nội; Xí nghiệp Bản đồ 1 - Bộ Quốc phòng, Trung tâm Khoa học tính toán trường Đại học Sư phạm Hà Nội đã góp phần vào thành công của Cuộc thi.
Cảm ơn phóng viên VTV2 Đài truyền hình Việt Nam, báo Giáo dục và Thời đại, các báo đài ở địa phương đã đến dự và đưa tin về Cuộc thi.
Xin cảm ơn tất cả và hẹn gặp lại tại Cuộc thi năm 2019.
TOÁN TUỔI THƠ
11
CUỘC THI CÂU LẠC BỘ TOÁN TUỔI THƠ TOÀN QUỐC 2018
CHILDREN'S FUN MATHS JOURNAL NATIONAL COMPETITION 2018
ĐỀ THI CÁ NHÂN BẬC TRUNG HỌC CƠ SỞ
SECONDARY SCHOOL INDIVIDUAL PAPER
Thời gian làm bài: 30 phút (Duration: 30 minutes)
Đề thi gồm 2 trang
Từ câu 1 đến câu 15 chỉ ghi đáp số, câu 16 viết lời giải đầy đủ vào mặt sau Tờ trả lời.
Question 1. Given x, y, z such that
X | N
2
=
y
113
Z
-=
5
and x + 3y + 6z = 82. Find M = x + y + z.
Question 2. The figure is a net of a cube. Which letter is opposite to H?
H
GM
PK
T
Question 3. Find n such that A = n3 - 2n2 + 2n-4 is a prime number. Question 4. How many integers x that satisfy both inequalities |x| +5<7 and |x −3|>2 ?
Question 5. Given the figure. ABC is a equilateral triangle, BCDE is a square and BEFGH is a regular pentagon. Find the measure of angle AHG.
G
H
?
F
E
A
B
C
D
Question 6. An bought a book with price of 50 000 VND. He gave the cashier 3 types of bank notes: 2000 VND, 5000 VND and 10 000 VND. The value of these notes is 50 000 VND and there are no more than 10 notes. How many bank notes of 2000 VND are there? Question 7. Find the positive integer n such that n + 1 and 4n + 29 are square numbers.
A
G
B
Question 8. How many line segments are there in the figure ?
E
F
D
H
C
12
Question 9. Denote: n! = 123. n. Find n such that in the prime factorisation of n!, the exponent of 2 is 11 and the exponent of 3 is 6.
Question 10. In the figure, ABC is equilateral triangle with side 3 cm. M is a point inside the triangle. Draw DE, FG, HI passing through M such that DE // AC; FG // AB; HI // BC (H, E = AB; G, D = BC; I, F e AC).
Find DE + FG+ HI.
Question 11. In the figure, triangle ABC has
AB = AC = 5 cm; BC = 8 cm. Find the length of the median BD of triangle ABC.
5 cm
B
E
A
M
H
LL
F
B
G
D
C
A
8 cm
D
C
Question 12. Find the sum of all roots of the equation: (3x-2) |x + 4| = x2 + 8x + 16.
=
Question 13. Given the figure such that
CACB = 6 cm; CD = 10 cm; DA DE = 8 cm.
B, C, D and E are collinear. Find the measure of angle BAE.
A
6
8
B 6 C
10
D
8
E
Question 14. How many square divisors of 86x920 x1018 are there?
Question 15. How many whole number less than 70 with 9 as the units digit is the sum of power of 2 and power of 3?
Question 16. (Written paper/Tự luận) Given triangle ABC with median AM and bisector AD. Let AC = 9 cm, AB = 6 cm. The area of triangle ABC is 24 cm2. Find the area of triangle ADM.
The diagrams are not drawn to scale
13
A
B
DM
C
CUỘC THI CÂU LẠC BỘ TOÁN TUỔI THƠ TOÀN QUỐC 2018
CHILDREN'S FUN MATHS JOURNAL NATIONAL COMPETITION 2018
ĐÁP ÁN ĐỀ THI CÁ NHÂN BẬC THCS
ANSWERS FOR SECONDARY SCHOOL INDIVIDUAL PAPER
Câu
Câu
Điểm
Điểm
Question 1. 20.
5 điểm
Question 9. n = 15.
5 điểm
Question 2. P.
5 điểm
Question 10. 6 cm.
5 điểm
Question 3. n = 3.
5 điểm
3
Question 4.2.
5 điểm
Question 11. BD =
√17 cm.
5 điểm
2
Question 5. 147°.
Question 12.-1.
5 điểm
5 điểm
Question 6. 5 notes.
5 điểm
Question 13. 135°.
5 điểm
Question 7. n = 35.
Question 8. 24.
5 điểm
Question 14. 3990.
5 điểm
5 điểm
Question 15. 4 numbers.
5 điểm
Question 16. 2.4 cm2.
5 điểm
Đáp án câu 16
A
Điểm
B
D M
C
Using the property of bisector in triangle, we have
=
DB
AB AC 6
DC DB
DC
9
5 điểm
Let
DB
6
=
DC
9
= k (with k > 0). We get DB = 6k, DC = 9k.
We have BC = DB + DC = 6k + 9k = 15k. We get BM
So DM = BM-BD
=
SADM DM
15k
2
―
3k 6k =
2
3k
1
We get
=
: 15k =
10
1
Therefore SADM
==
SABC
=
.24 = 2.4 (cm2).
10
10
SABC BC 2
1
Học sinh làm cách khác đúng vẫn cho điểm tối đa.
14
=
1BC
15k
=
5 điểm
2
2
5 điểm
5 điểm
5 điểm
CUỘC THI CÂU LẠC BỘ TOÁN TUỔI THƠ TOÀN QUỐC 2018
CHILDREN'S FUN MATHS JOURNAL NATIONAL COMPETITION 2018
ĐỀ THI BẬC TRUNG HỌC CƠ SỞ (SECONDARY SCHOOL PAPER) VÒNG 1: TIẾP SỨC TOÁN (ROUND 1: RELAY RAGE)
Thời gian: 30 phút cho cả 6 câu hỏi (Duration: 30 minutes for 6 problems)
Problem 1. Given the non-zero real numbers
a, b such that 4a2 5ab + b2 = 0. Find the
value of C =
―
ab+8a2
4a2 -b2
Problem 2. Given that B
with -3 ≤x≤1.
=
4x2 + 18x + 25
a) Find the minimum value of B. b) Find the maximum value of B.
Problem 3.
B
D
9 cm
45°
45°
A
12 cm
In the figure, triangle ABC has AB
C
=
9 cm;
AC 12 cm. Point D is on BC such that
=
13
||
3y 6z
2
9 30
82
=
2
41
||
2
x + 3y+6z
2+9+30
X | N
2
=
y
3
=
Z x + y + z
=
5
10
⇒M = x + y + z = 20.
=
= 2
Bài 2. Ta thấy mặt chứa chữ P kề với mặt chứa các chữ
H
GM
M, K, T, G. Do đó mặt chứa
chữ H đối diện với mặt chứa chữ P.
PK
T
Bài 5.
G
LL
F
H
?
ヨ
A
B
C
D
Bài 3. Ta có
A = n3 - 2n2 + 2n - 4 = (n-2)(n2 + 2). Vì A là số nguyên tố nên n − 2 >0. Suy ra n – 2 = 1 và n + 2 là số nguyên tố.
Vậy n = 3.
Bài 4. Ta có
x+5<7-22
X>5
|x − 3 >2 ⇒
⇔
X-3 <-2 x<1.
Vì ABC là tam giác đều, BCDE là hình vuông, BEFGH là ngũ giác đều nên
ABC = 60°; EBC = 90°; HBE = BHG = 108°.
Suy ra
ABH = 360° - 60° – 90° – 108° = 102°.
Do đó AHB = 39°
Vậy AHG= 39° +108° = 147
Bài 6. Gọi số tờ tiền 2000 đồng, 5000 đồng
và 10000 đồng thứ tự là a, b, c (a, b, c = N*,
a, b, c < 10).
Ta có 2000a + 5000b + 10000c = 50000
Các số nguyên x thỏa mãn cả hai bất đẳng Suy ra 2a + 5b + 10c = 50.
thức là −1, 0.
Vậy có 2 số nguyên thỏa mãn cả hai bất
đẳng thức trên.
a=67
n=35
20
Do đó 2a : 5.
Mà (2, 5) = 1 nên a : 5, tức là a = 5.
Vậy có 5 tờ tiền 2000 đồng.
Bài 7. Đặt n + 1 = a, 4n + 29 = b (a, b c N).
Ta có b2 – 4 = 25.
Suy ra (b - 2a)(b + 2a) = 25.
Mà b +2a>0 nên b – 2a>0 và b+2a>b−2a nên suy ra b – 2a = 1, b + 2a = 25.
Do đó a = 6.
=
Vậy n = 35.
Bài 8.
A
G
B
E
O
F
Bài 11.
B
5 cm
A
D
G
M
8 cm
C
D
H
C
Xét 8 đường thẳng AB, BC, CD, DA, AC, BD, EF và GH. Trên mỗi đường thẳng đó có 3 đoạn thẳng. Vậy số đoạn thẳng trên hình vẽ là 8.3 = 24 (đoạn thẳng).
Bài 9. Có 5 bội nhỏ nhất khác 0 của 3 là 3, 6, 9, 12 và 15 (trong đó 9 = 3). Do đó n > 15.
Ta thấy 16! có các thừa số chẵn là 2, 4, 6, 8, 10, 12, 14, 16.
Vẽ đường trung tuyến AM của tam giác ABC, gọi G là giao điểm của AM và BD.
Ta có G là trọng tâm tam giác ABC và BM =
8:24 cm.
Áp dụng định lí Pythagoras ta có AM2 = AB2 - BM2 52 - 42 = 32.
=
Suy ra AM = 3 cm, từ đó GM = AM : 3 = 3:3
= 1 cm.
Do đó BG = BM2 + GM2 = 4 + 12 = 17, từ đó BG = √17 cm.
3
Suy ra 16! khi phân tích ra thừa số nguyên tố Vậy BD=-BG=
thì số mũ của 2 là 14.
Do đó n < 16.
Vậy n = 15.
Bài 10.
A
LL
F
3
2BG=√17 cm.
Bài 12. Phương trình tương đương với
(3x − 2)x + 4 = (x + 4)2
x+4=0
3x-2=x+4
x = -4 X
X = 3
Vậy tổng hai nghiệm của phương trình là (-4)+ 3 = (-1).
B
E
H
X
G
X
y
M
y
Z
N
D
C
Dễ dàng chứng minh được các tam giác EHM, FMI và MGD là các tam giác đều.
Đặt HM = EM
=
EH = x, FM = MI =
FI = y,
Bài 13.
A
MG = MD = GD = z.
Ta có 3(x + y + z) = 9, từ đó x +y+z=3. Suy ra
6
8
DE + FG+HI = 2(x + y + z) = 2.36 cm.
B 6 C
10
D 8
E
21
+
VÌ AC2 + AD2 = 6 + 8 = 102 = CD nên theo định lí Pytagoras đảo thì tam giác ACD vuông tại A.
Vì các tam giác ABC và ADE thứ tự cân tại C
và D nên ACD = 2BAC; ADC = 2DAC.
Bài 16. Theo tính chất đường phân giác trong tam giác ta có
DB DC DB DC
=
AB AC
6 9
A
1
Suy ra BAC +DAE =
——
(ACD + ADC)
2
1
.90° = 45°.
2
Vậy BAE = 90° +45° = 135°
Bài 14. Ta có
86×920 ×1018 = (23)6 ×(32)20 ×(2.5)18
=
= 236 × 340x518
:
Các ước chính phương của số
B
D M
C
DB DC
Đặt
= k (với k > 0).
6
9
Suy ra DB = 6k, DC = 9k.
86×920 ×1018 là bình phương ước của số Suy ra DB =
218 x 320 × 59. x320x59.
Vậy số các ước chính phương của số gô ×920 ×1018 là
(18 + 1)(20+ 1)(9 + 1) = 3990.
Ta có BC = DB + DC = 6k + 9k = 15k.
Do đó BM
1 = -BC
2
==
15k
2
15k
3k
Suy ra DM =BM-BD
=
6k:
-
2
2
Bài 15. Các lũy thừa của 2 nhỏ hơn 70 là 1,
Từ đó suy ra
2, 4, 8, 16, 32 và 64.
Các lũy thừa của 9 nhỏ hơn 70 là 1, 3, 9 và
SADM SABC
DM
3k
1
=
:15k =
BC
2
10
1
1
Vậy SADM
=
SABC
10
=
10
.24 = 2.4 (cm2).
27.
Lập bảng
Lũy thừa
Lũy thừa
Các tổng có tận
của 2
của 3
cùng bằng 9
1
2, 32
27
29, 59
4, 64
16
3
19
8
1
9
9
Vậy tất cả có 4 SỐ.
22
CÂU LẠC BỘ
ĐỀ THỊ
TOÁN TUỔI THƠ CÂU LẠC BỘ TTT
NGUYỄN ĐỨC TẤN
DƯƠNG THU TRANG (dịch)
Kì 18
12
=
CLB1. Given a, b, c such that a3 + b3 + c3 a+b+c=0. Find the value of M = abc. CLB2. Let a, b, c, d be the integers such that |ab|+|bc|+|cd| + |da| = a2018 + 2019. Find the remainder when a12 divided by 16. CLB3. The total ages of father and son is 49. In 3 years' time, son's age is 37.5% of father's age. How many years ago such that father's age is 3.5 times son's age?
CLB4. Let ABCD be a square with side of 12 cm. E is on AD such that DE = 7 cm. Find the position of M on diagonal AC such that the sum of MD + ME is smallest. Find this smallest value. CLB5. ABC is a equilateral triangle with D on AC
such that CD
=
1
AC. The perpendicular bisector
3
JAC
of BD intersects AB, BC at E, F respectively. Find
2
2
2
2017
1
1
1-
=
2.3
3.4
n. (n + 1)
6045
1.4 2.5 3.6
2.3 3.4 4.5
(n-1)(n+2) 2017
1.2.3...(n-1) 4.5... (n+2)
2.3...n
n+2 2017
=
3n 6045
CLB4. Ta có
ab + 1 bc +1
b
Do đó a-b
b-c
=
n(n + 1)
6045
2017
3.4...(n + 1)
6045
⇒n=2015.
=
ca +1
1
⇒a+
= b+
C
a
b
1
1
b-c
==
C b
bc
1
1 c-a
=
;c-a
a
C
ac
(a - b)(b−c)(c− a) -
=
=
110
1 =C+-
с а
1 1 a-b
=
b a ab
(a - b)(b−c)(c–a)
a2b2c2
= c
Mà abc = 1. Suy ra a = b hoặc b = c hoặc c = a. Do đó a = b = C.
CLB5. • TH1. E nằm trong tam giác ABC.
A
|||
B
E
D
C
D
A
/m
E
B
C
LL
F
the ratio
SADE SCDF
Kết quả
Kì 16 (TTT2 số 183)
CLB1. Ta có pq = (2r2 + 4) : 2
Suy ra p = 2 hoặc q = 2.
r2
• Xét p = 2, ta có q - r = 2 → q = 2 +2.
* r
* Nếu r = 3 thì q = 11 là số nguyên tố.
* Nếu r = 3 thì r2 chia cho 3 dư 1.
Suy ra q = (r + 2) : 3. Mà r+2>3 nên q là hợp số.
=
Xét q = 2, làm tương tự ta cũng có p 11. Vậy p = 2; q = 11; r = 3 hoặc p = 11; q = 2; r = 3. CLB2. Ta có (a + b + c)(ab + bc + ca) = abc. ← (a + b)(ab + bc + ca) + c2(a + b) = 0. ← (a + b)(ab + bc + ca + c2) = 0.
↔ (a + b)(b + c)(c + a) = 0.
→a+b = 0 hoặc b + c = 0 hoặc c + a = 0.
• Nếu a + b = 0, từ đó a = −b.
Suy ra (a + b + c)2017 = a2017 + b2017 +c2017 Do đó M = 1.
• Nếu b + c = 0 hoặc c + a = 0, làm tương tự ta được M= 1.
CLB3. Ta có
F
Ta có ADC = EDF (= 90°) → ADE = CDF.
VÌ ADAE = ADCF (c.g.c) nên AED = CFD. Ta có
AED + FEC = AEC - DFE = 140° - 45° = 95°
→ CFD + FEC = DFE + EFC + FEC = 95°
EFC+FEC = 95° - DFE = 50°
ECF = 180° -50° = 130°.
• TH2. E nằm trong tam giác ADC. Làm tương tự như trên ta được ECF = 50°.
HH
SINCE 1959
Lên truyền
CTY CP VĂN PHÒNG PHẨM
Nhận xét. Các bạn sau có lời giải tốt . HỒNG HẢ được thưởng kì này: Hoàng Vũ Nghị, 8E, THCS Vĩnh Tường, Vĩnh Tường, Vĩnh Phúc; Nguyễn Thị Diệu Linh, 8I, THCS Văn Lang, TP. Việt Trì, Phú Thọ; Trần Trọng Quang Huy, 8A, THCS Trần Huy Liệu, Vụ Bản, Nam Định; Nguyễn Thu Hiền, 8A3, THCS Thị trấn Kỳ Sơn, Kỳ Sơn, Hòa Bình.
NGUYỄN NGỌC HÂN
23
GIẢI TOÁN THẾ NÀO
Ko
SỬ DỤNG TÍNH CHẤT ĐƯỜNG TRUNG TUYẾN ỨNG
VỚI CẠNH HUYỀN CỦA TAM GIÁC VUÔNG
THÁI NHẬT PHƯỢNG
(GV. THCS Nguyễn Văn Trỗi, Cam Nghĩa, Cam Ranh, Khánh Hòa)
hi giải các bài toán hình học liên quan đến tam giác vuông với các bạn học sinh lớp 7. Nếu sử dụng bổ đề về mối quan hệ giữa đường trung tuyến ứng với cạnh huyền và cạnh huyền của tam giác vuông đó thì các bạn sẽ tìm được lời giải khá ngắn gọn cho các bài toán.
Bổ đề. Đường trung tuyến ứng với cạnh huyền của một tam giác vuông bằng nửa cạnh huyền.
Chứng minh. Xét AABC vuông tại A với đường trung tuyến AM, ta sẽ chứng minh
BC
AM
2
B
1
A
D
M 2 1
1
C
Thật vậy, trên tia đối của tia MA lấy điểm D sao cho MD = MA.
Vì MC = MB, M =M2 nên AMDC = AMAB
(c.g.c).
Từ đó suy ra DC = AB và C1 = B1.
Do đó CD II AB.
Ta lại có AC I AB nên AC l CD.
Xét tam giác vuông ACD và tam giác vuông CAB có:
CD = AB, AC chung nên AACD = ACAB.
Suy ra AD = BC.
Kết hợp với AM =
AD
2
ta được AM =
BC
2
Bài toán 1. Cho tam giác ABC vuông tại A với AB = 9 cm, AC = 12 cm. Gọi G là trọng tâm của tam giác ABC. Tính độ dài đoạn thẳng AG. Lời giải.
B
A
G
M
Áp dụng định lí Pythagoras ta có BC2 = AB2 + AC2 = 92 + 122 = 225.
Suy ra BC = 15 (cm).
Gọi M là trung điểm của BC.
Áp dụng bổ đề trên ta có
1
AM =-BC= 7,5 (cm).
2
Do đó AG =
2AM = 5 (cm).
3
C
Bài toán 2. Cho tam giác ABC với M, N thứ tự là trung điểm của AB, AC. Chứng minh rằng MN // BC.
Lời giải.
M
A
B
H
24
N
C
Kẻ AH L BC tại H.
Ta có HM, HN thứ tự là các đường trung tuyến ứng với cạnh huyền của các tam giác vuông AHB và AHC nên theo bổ đề trên ta
có MH =
AB
2
AC
=
AM; NH =
2
= NA.
Do đó MN là đường trung trực của AH.
Suy ra MN L AH, kết hợp với BC – AH ta có MN // BC.
Bài toán 3. Cho tam giác ABC không vuông với hai đường cao BD và CF. Gọi M, N thứ tự là trung điểm của BC và DF. Chứng minh rằng MN vuông góc với DF.
Lời giải.
F
LL
A
N
D
Suy ra C, A, D thẳng hàng.
Từ đó ADEA = ABCA (c.g.c) Do đó DE = BC. (1)
Gọi chân đường vuông góc từ A đến BC là H. Vẽ DT I MA, EK I MA.
Ta có AD = AB và DAT = CAH = ABH
ABHA (cạnh huyền - góc
Suy ra AATD = nhọn).
Do đó DT = AH.
Chứng minh tương tự ta có AAEK = ACAH nên EK = AH.
Ta có DT
TDM = KEM
=
EK (vì cùng bằng AH);
Suy ra ADTM = AEKM (g.c.g).
Từ đó MD = ME.
B
M
C
Ta có FM, DM thứ tự là các đường trung tuyến ứng với cạnh huyền BC của các tam giác vuông BFC và BDC.
Theo bổ đề trên ta có DM
BC
2
= FM.
Từ đó ADMF cân tại M, kết hợp với ND = NF, suy ra MN | DF.
Bài toán 4. Cho tam giác ABC vuông tại A. Bên ngoài tam giác ABC dựng các tam giác BAD, CAE là các tam giác vuông cân tại A. Đường thẳng qua A và vuông góc với BC cắt
DE tại M. Chứng minh rằng AM =
B
BC
2
Theo bổ đề ta có AM
=
DE
(2)
2
BC
2
Từ (1) và (2) suy ra AM =
Bài tập vận dụng
Bài 1. Cho AABC vuông tại A và C =30° Gọi M là trung điểm của BC. Chứng minh rằng AABM là tam giác đều.
Bài 2. Cho AABC vuông tại A với BC = 10 cm. Tìm giá trị lớn nhất của diện tích AABC.
Bài 3. Cho AABC và hai đường cao BH, CK. Chứng minh rằng B, K, H, C cùng thuộc một đường tròn.
Bài 4. Cho AABC vuông tại A, đường cao
BC
2
AH. Chứng minh rằng AHs
Khi nào
Lời giải.
H
D
C
K
A
TM
E
Ta có AC L AB và AD LAB.
AH=
=
BC
-? 2
B
25
COMPA VUI... TỈNH
Kết quả
(KI NÀY VẼ THẾ NÀO NHỈ?
Kì này
Bài toán. Cho hai đường thẳng a và b cắt nhau ngoài tờ giấy và điểm M nằm trong tờ giấy. Chỉ dùng thước thẳng (không có vạch chia), hãy vẽ qua M một đường thẳng đi qua giao điểm của a và b.
(TTT2 số 183)
CÓ VẼ ĐƯỢC KHÔNG?
a) Giả sử có m đoạn thẳng và mỗi đoạn thẳng cắt đúng n đoạn thẳng khác với điều kiện mỗi giao điểm chỉ thuộc hai đoạn thẳng thì số giao
mn
điểm là (do đếm số giao điểm thuộc mỗi
2
đoạn thẳng n lần và mỗi giao điểm được đếm hai lần). Do số giao điểm là số nguyên nên tích mn phải là số chẵn. Như vậy không thể sắp xếp 7 đoạn thẳng sao cho mỗi đoạn thẳng cắt đúng 3 đoạn thẳng khác với điều kiện mỗi giao điểm chỉ thuộc hai đoạn thẳng. b) Nếu mỗi giao điểm là điểm chung của hai hay nhiều đoạn thẳng thì có thể sắp xếp được như ở Hình 1 và Hình 2.
NGUYỄN XUÂN BÌNH (Hà Nội)
ĐƯỢC THƯỞNG KÌ NÀY Thi giải toán qua thu
Vũ Minh Nguyệt, 6E, THCS Vĩnh Tường, Vĩnh Tường, Vĩnh Phúc; Hoàng Tuấn Dũng, 7A1, THCS Minh Khai, TP. Hà Giang, Hà Giang; Nguyễn Thu Hiền, 8A3, THCS Thị trấn Kỳ Sơn, Kỳ Sơn, Hòa Bình; Ngô Bảo An, Nguyễn Lê Tùng Dương, 9A0, THCS Nguyễn Trường Tộ, Đống Đa, Hà Nội; Trần Cao Bảo Châu, 7A, THCS Trần Hưng Đạo, Buôn Ma Thuật, Đắk Lắk; Huỳnh Nguyên Phúc, 9A1, THCS Mỹ Lộc, Phù Mỹ, Bình Định; Nguyễn Thị Quỳnh Chi, 8A1, THCS Yên Phong, Yên Phong, Bắc Ninh; Trần Phương Mai, 7B, THCS Hồ Xuân Hương, Quỳnh Lưu, Nghệ An; Trương Ngọc Tâm, 8D, THCS Nhữ Bá Sỹ, thị trấn Bút Sơn, Hoằng Hóa, Thanh Hóa; Phùng Đăng Dương, 7C, THCS Văn Lang, TP. Việt Trì, Phú Thọ.
Hình 1
Hình 2
Nhận xét. Rất tiếc kì này không có bạn nào giải đúng. Quà tặng đành... để lại kì sau. Các bạn cố gắng lên nhé!
ANH COMPA
26
(ITA) ĐIỀN SỐ THÍCH HỢP
Kì này
Bài 1. Điền số thích hợp vào ô trống
5 x 4
6
2+6
DO TRÍ THÔNG MINH
2 x 3
2
1+ 1
3 × 7
8
4+1
8 x 2
4+2
Kết quả
NGUYỄN ĐỨC TẤN (TP. Hồ Chí Minh)
TÌM SỐ CÒN THIẾU (TTT2 số 183)
Bài 1. Trong mỗi hình, số ở dưới bằng bình phương của số ở bên trái trừ đi tổng các chữ số của số ở bên phải. Vậy số thích hợp điền vào chỗ (?) là 32 – (9 + 0) = 0.
Bài 2. Các số trong bảng là các số được viết theo thứ tự ngược lại của các số chính phương gồm hai chữ số. Như vậy số điền vào ô (?) là số viết ngược lại của số 64, tức là số 46. Nhận xét. Các bạn cần phân biệt số được viết theo thứ tự ngược lại của số ab (là số ba) với số nghịch đảo của số này.
HH
SINCE 1959
CÁCH VĂN PHÒNG PHẨM
HONG HA
lưu truyền thống - (Viết tương lai
Xin trao thưởng cho các bạn: Nguyễn Đức Bảo
Châu, Đào Xuân Thao, 7A2, THCS Yên Phong,
Yên Phong, Bắc Ninh.
NEW
NGUYỄN XUÂN BÌNH
te
46
THÔNG BÁO CHUYỂN TRỤ SỞ
Từ tháng 7 năm 2018, tòa soạn tạp chí Toán Tuổi thơ chuyển trụ sở về địa điểm mới. Địa chỉ liên hệ và gửi bài:
TẠP CHÍ TOÁN TUỔI THƠ
Tầng 2, nhà A, số 187B Giảng Võ, P. Cát Linh, Q. Đống Đa, Hà Nội
Điện thoại: 024.3568 2701; Fax: 024.3568 2702.
Email: bbttoantuoitho@gmail.com; tapchitoantuoitho@gmail.com
Website: www.toantuoitho.vn
Trân trọng thông báo để các độc giả được biết và tiện giao dịch.
TOÁN TUỔI THƠ
27
ĐỀ THI TUYỂN SINH LỚP 10 TP. HÀ NỘI
Bài 1. (2 điểm)
Năm học 2018 - 2019 Môn thi: Toán (chuyên Toán) Thời gian làm bài: 150 phút
1. Giải phương trình x + 3x + 8 = (x + 5)x2 +x+2
2. Giải hệ phương trình
Bài 2. (2,5 điểm)
ly2
2 -2xy = 8x2-6x+1
2 3
|y2 = x3 +8x2 -x+1.
1) Cho p, q là hai số nguyên tố lớn hơn 5. Chứng minh p^ + 2019q4 chia hết cho 20. 2) Cho các số nguyên dương a, b, c, d thỏa mãn a b + c.
b) Chứng minh a là một số chính phương.
Bài 3. (1,5 điểm)
1) Với x, y, z là các số thực thỏa mãn xyz = 1, chứng minh
1
+
+
1
1
= 1. xy+x+1 yz+y+1 ZX + Z+1
2) Với x, y, z là các số thực dương thay đổi và thỏa mãn
biểu thức P =
Bài 4. (3 điểm)
1
+
1
1
2
1
1 1
——
= 3, tìm giá trị lớn nhất của
+ + X y Z
√2x2 + y2+3 √2y2+z2 +3 √2z2 + x2 +3
Cho tứ giác ABCD (không có hai cạnh nào song song) nội tiếp đường tròn (O). Các tia BA và CD cắt nhau tại điểm F. Gọi E là giao điểm của hai đường chéo AC và BD. Vẽ hình bình
hành AEDK.
1) Chứng minh tam giác FKD đồng dạng với tam giác FEB.
2) Gọi M, N tương ứng là trung điểm của các cạnh AD, BC. Chứng minh đường thẳng MN đi qua trung điểm của đoạn thẳng EF.
3) Chứng minh đường thẳng EF tiếp xúc với đường tròn ngoại tiếp của tam giác EMN.
Bài 5. (1 điểm)
Cho tập hợp S = {xeZ|1≤x≤50}. Xét A là một tập hợp con bất kì của tập hợp S và có tính
chất: Không có ba phần tử nào của tập hợp A là số đo độ dài ba cạnh của một tam giác vuông.
1) Tìm một tập hợp A có đúng 40 phần tử và thỏa mãn điều kiện đề bài.
2) Có hay không có một tập hợp A có đúng 41 phần tử và thỏa mãn điều kiện đề bài? Hãy giải thích câu trả lời.
ĐẶT MUA TẠP CHÍ CẢ NĂM HỌC TẠI CÁC CƠ SỞ BƯU ĐIỆN TRONG CẢ NƯỚC
MÃ ẤN PHẨM: C 169.1
28
ĐỀ THI TUYỂN SINH LỚP 10 TP. HÀ NỘI
Bài 1. (2,0 điểm)
Năm học 2018 - 2019 Môn thi: Toán (chuyên Tin)
Thời gian làm bài: 150 phút
a) Giải phương trình x2 + 2x + 7 = (x + 3)x2 +5.
b) Giải hệ phương trình
Bài 2. (2,5 điểm)
(xy)(x2 y2)=1
(x + y)(x2 + y2) = 1.
a) Tìm tất cả cặp số nguyên (x; y) thỏa mãn 4x
b) Cho hai số nguyên dương a, b thỏa mãn 3a
chính phương.
Bài 3. (1,5 điểm)
+ 8xy + 3y +2x+y+2=0.
-
+ a=4b2 _b. Chứng minh a + b là một số
a) Với x, y, z là các số thực thay đổi và thỏa mãn xyz = 1, chứng minh
1
+
1
1
+
=
1.
xy+x+1 yz + y + 1 ZX+z+1
b) Với x, y, z là các số thực dương thay đổi và thỏa mãn xyz ≥ 1, tìm giá trị lớn nhất của biểu
thức P
=
1
+ √xy + x + 1
1
yz+y+1
1
+
ZX + Z+1
Bài 4. (3,0 điểm)
Cho tam giác nhọn ABC cân tại A, đường cao BE và nội tiếp đường tròn (O; R). Kẻ đường kính BD của đường tròn (O). Đường thẳng BE cắt các đường thẳng AD và AO lần lượt tại các điểm I và H.
a) Chứng minh BHBI= 2R2.
b) Gọi M là trung điểm của cạnh AB. Lấy điểm N thuộc tia đối của tia OA sao cho ON=
Chứng minh tứ giác AMNC là tứ giác nội tiếp.
R
2
c) Gọi K là trung điểm của cạnh BC. Chứng minh đường thẳng KE đi qua trung điểm của đoạn thẳng OI.
Bài 5. (1,0 điểm)
Trên một đường tròn cho 2018 điểm phân biệt. An và Bình cùng chơi trò chơi như sau: Mỗi lượt chơi, một bạn sẽ nối 2 điểm trong 2018 điểm đã cho để được một dây cung sao cho dây cung vừa được vẽ không có điểm chung với bất kì dây cung nào đã vẽ trước đó. Hai bạn luân phiên thực hiện lượt chơi của mình. Bạn đầu tiên không thể thực hiện được lượt chơi của mình là người thua cuộc. Nếu An là người đi trước, hãy chỉ ra chiến thuật chơi để An luôn là người thắng cuộc.
29
NHÌN ĐÔ THẾ GIỚI
MỘT GÓC NHÌN HAI BÀI HÌNH TRONG ĐỀ THI IMO 2018
ThS. NGUYỄN BÁ ĐANG (Hà Nội)
ì thi Olympic Toán học Quốc tế (International Mathematical Olympiad viết tắt là IMO) được tổ chức lần thứ nhất tại Romania có 7 nước tham gia (các nước Đông Âu), từ năm 1970 mở rộng thành kì thi toán quốc tế dành cho học sinh Trung học. Năm 2018 Romania là nước đăng cai tổ chức kì thi IMO lần thứ 59, có trên 100 nước tham dự, mỗi nước không quá 6 học sinh, đoàn Việt Nam đoạt một Huy chương Vàng, hai Huy chương Bạc, ba Huy chương Đồng. Tôi muốn giới thiệu hai bài hình phẳng mà học sinh lớp 9 với học lực khá làm được, bài 1 có gần 400 thí sinh giải được đầy đủ; bài 6 được coi là khó nên chỉ có 18 thí sinh làm trọn vẹn. Để giải bài 6 cần bổ sung thêm kiến thức về đường tròn Apollonius.
Bài 1. Cho tam giác nhọn ABC. Các điểm D và E lần lượt nằm trên các đoạn thẳng AB và AC thỏa mãn AD = = AE. Đường trung trực của BD cắt cung nhỏ AB tại F, đường trung trực của CE cắt cung nhỏ AC tại G. Chứng minh rằng các đường thẳng DE và FG song song (hoặc cùng nằm trên một đường thẳng). Lời giải. Các đường thẳng FD, GE cắt đường tròn ngoại tiếp tam giác ABC thứ tự tại các điểm P và Q (Hình 1).
Vì F nằm trên đường trung trực của BD nên
FBD = FDB = ADP.
Mà ABF = APF (cùng chắn cung AF).
Suy ra ADP = APF = APD.
30
Do đó tam giác ADP cân tại A, từ đó AP = AD.
A
P
E
G
D
F
B
Hình 1
C
Theo giả thiết thì AD = AE nên P thuộc đường tròn tâm A bán kính AD. Chứng minh tương tự Q thuộc đường tròn tâm A bán kính AD. Suy ra P, E, D, Q cùng thuộc một đường tròn. 1) Nếu 3 điểm P, D, E thẳng hàng thì E trùng với P (do APD = ADP = AED ). Do đó E trùng với C và G trùng với C, suy ra DE và FG cùng nằm trên một đường thẳng.
2) Tương tự, nếu ba điểm Q, D, E thẳng hàng thì D, Q trùng với B nên DE và FG cùng nằm trên một đường thẳng.
3) Giả sử D, E, P, Q không thẳng hàng thì
PDE = PQE = PQG = PFG.
Suy ra PDE =PFG, từ đó DE II FG.
Bài 2. Cho tứ giác ABCD thỏa mãn AB.CD=BC.DA . Điểm X nằm trong tứ giác
ABCD sao cho XAB = XCD và XBC = XDA .
Chứng minh rằng BXA + DXC = 180°
Để giải bài này ta cần kiến thức bổ sung về (Xét tương tự khi ABMX và DXMC là các tứ đường tròn Apollonius. (Hình 2)
M
J
B
Ο Α
|
giác nội tiếp. Khi hai đường tròn (XAB) và (XCD) tiếp xúc nhau tại X thì từ XAB = XCD, xét hai góc đỉnh X tạo bởi tiếp tuyến và dây cung XB, XD suy ra D, X, B thẳng hàng).
A
M
D
B
О
Hình 2
Cho đoạn thẳng AB = a và k là số thực cho
trước (0 AD thì CD < BC, xét tương tự).
Suy ra A và C nằm trên đường tròn Apollonius với tâm O nằm trên BD và OA là tiếp tuyến của đường tròn (ABD), OC là tiếp tuyến của đường tròn (BCD).
Ta có AOC = ABC – BAO-BCO
= ABC - ADO - CDO = ABC - ADC
= ABX + XBC - ADC
= 180° - AMX + XDA – ADC
=
= 180° - AMX - XDC
= 180o – AMX – XMC = 180° – AMC.
=
Từ đó AOC+ AMC = 180°.
Suy ra tứ giác AOCM nội tiếp.
VÌ OA = OC nên AMO = OMC.
Suy ra BXA = AMB = 180° - AMD
=
= 180° - CMD = 180° - DXC
→ BXA + DXC = 180°.
A
COMPAVUI... TÍNH
Ô chữ BIRDS
HOÀNG THỊ PHƯỢNG
(GV. TH Ngô Đức Kế, Can Lộc, Hà Tĩnh)
Các loài chim đang “ẩn trốn” trong ô chữ này. Chúng mình cùng tìm ra
tên của các loài chim đó nhé!
THARK SWANUNA
AAAPMINOON MK
WWHMOPATPAUT
SKOOWLFOE ASS
TDAEQSPMLUD T
ASGLSAEOIFMO
LAKEHHAUCHER
BACKEE CLASNK
ACRDREORNUE A
TOAFOLCEUS PC
RFNAN SKSLCAH
OHEEPTKALARA
SUYCIE EME DRL SAXVDRAUTWOO
ROVOLLEL BATT
Kết quả APRIL SHOWERS (TT2 số 183)
Chủ Vườn đã nhận được khá nhiều bài của các bạn gửi về. Mặc dù một số bạn đặt câu còn chưa thực sự đúng ngữ pháp hoặc viết sai chính tả nhưng Chủ Vườn thấy rất vui mừng vì có nhiều bạn “đam mê” tiếng Anh cũng như đã dành thời gian ghé qua Vườn Anh chơi. Sau đây là đáp án kì trước:
Từ láy pitter patter nghĩa là tiếng lộp độp, lộp cộp, rộn rã. Cách viết khác của từ này là pit-a-pat.
Còn đây là hai trong số nhiều ví dụ của các bạn có sử dụng từ pitter patter
I heard the pitter patter of my mother's footsteps on the stairs.
HH
SINCE 1959
G CỤ VĂN PHÒNG PHẨM
HONG HA
Lưu truyền thống - Viết tương lai
Phần thưởng kì này sẽ được gửi tới
các bạn sau: Đỗ Hồng Liên, 8A3, THCS Trưng Vương, Mê Linh, Hà Nội; Mẫn Mai Phương, Nguyễn Phương Linh, 6A1, THCS Yên Phong, Yên Phong, Bắc Ninh; Trần Trọng Quang Huy, 8A, THCS Trần Huy Liệu, Vụ Bản, Nam Định; Vũ Huyền Trang, 8H, THCS Văn Lang, TP. Việt Trì, Phú Thọ; Cao Thị Khánh Linh, 8B, THCS Hoàng Xuân
Her heart went pitter patter as she opened Hãn, Đức Thọ, Hà Tĩnh.
the letter.
Chủ Vườn
32
Kết quả
Giải toán qua thư
Bài 1(183). Với mỗi số nguyên dương a, kí hiệu S(a) là tổng tất cả các chữ số của số a. Hãy tìm giá trị nhỏ nhất của S(4020n) với n là số nguyên dương.
Lời giải. Ta có 4020:3→4020n:3
HAC 90° - ACH-90° -30° = 60°.
=
Do đó MAN = MAH = HAB
1
=
•
2
60° = 30°.
Suy ra BAC = 90° → AABC vuông tại A.
=S(4020n):3= S(4020n)= {3; 6; 9; 12; ...}. Nhận xét. Tam giác vuông ABC có
Ta nhận thấy với n = 5 thì
4020.n=4020.5=20100. Mà S(20100) = 3. Vậy giá trị nhỏ nhất của S(4020n) là 3 tương ứng với n = 5.
Nhận xét. Bài toán sử dụng dấu hiệu “một số chia hết cho 3 thì tổng các chữ số của nó cũng chia hết cho 3”. Xin được khen bạn đã có lời giải đúng: Vũ Minh Nguyệt, 6E, THCS Vĩnh Tường, Vĩnh Tường, Vĩnh Phúc.
PHÙNG KIM DUNG Bài 2(183). Cho tam giác ABC với AB < AC, đường cao AH, trung tuyến AM. Biết rằng BAH = HAM = MAC. Chứng minh rằng tam giác ABC là tam giác vuông.
Lời giải. Ta có AABH = AAMH (g.c.g).
1
Suy ra HB = HM= -MB.
2
Kẻ MN L AC (N = AC).
Khi đó ta có AANM = 1AHM.
Suy ra MN = MH = 1MB=1MC.
B
=
A
H
2
M
2
N
Trong tam giác vuông MNC có
MN=-MC → NCM = 30°
1
2
C
ACB = 30°, ABC=60° nói trên còn được gọi là nửa tam giác đều. Số bài giải gửi về tòa soạn tương đối nhiều. Các bạn sau có lời giải gọn hơn cả: Lê Đăng Quang, 7A1, THCS Yên Phong, Yên Phong, Bắc Ninh; Hán Vũ Long, Trần Anh Vũ, Phùng Đăng Dương, 7C, THCS Văn Lang, TP. Việt Trì, Phú Thọ; Nguyễn Thanh Tiến, Nguyễn Thị Hiền Diệu, Lê Minh Long, Nguyễn Quang Đức, Trịnh Duy Đạt, Lê Đức Chính, 7B, THCS Nhữ Bá Sỹ, Hoằng Hóa, Thanh Hóa; Phạm Ngọc Trinh, Trần Phương Mai, 7B, THCS Hồ Xuân Hương, Quỳnh Lưu; Trần Thiên Ngân, Lưu Bảo Phúc, Phạm Thị Khánh Huyền, Bùi Đoàn Hà Trang, Trần Trọng Huy, 7A, THCS Cao Xuân Huy, Diễn Châu; Nguyễn Lê Hưng, Đinh Thị Việt Hà, Trần Tùng Chi, Nguyễn Trần Khánh Huyền, Võ Văn Hải, 7D, THCS Đặng Thai Mai, TP. Vinh; Phùng Lê Anh Đức, 7A1, THCS Hải Hòa, thị xã Cửa Lò, Nghệ An; Hồ Xuân Hiệp, 7A, THCS Lê Bình, Hương Sơn, Hà Tĩnh.
HỒ QUANG VINH
Bài 3(183). Giải hệ phương trình
4√3x+4y+ √√8-x+y=23
3√8-x+y-2√38+6x-13y = 5.
Lời giải. ĐKXĐ 3x + 4y > 0; 8−x+y>0; 38+6x-13y >0.
Đặt a =
C=
√3x+4y; b = √8-x+y;
38+6x-13y; a, b, c ≥0.
33
Hệ phương trình đã cho trở thành
4a=23-b
2c=3b-5
2
(1)
(2)
a2+9b2 + c2 = 110 (3)
Thay (1) và (2) vào (3) ta được
2
2
23-b
+9b2+
3b-5
= 110
4
2
181b2-166b-1131-0
-377
→b=3 (thỏa mãn) hoặc b =
181
(loại).
Từ đó giải được a = 5. Vậy hệ phương trình có nghiệm duy nhất (x; y) = (3; 4).
Nhận xét. Sau khi tìm ra mối liên hệ giữa ba căn thức, bài toán được giải bằng phương pháp thế quen thuộc. Hai bạn sau có lời giải tốt: Nguyễn Thị Quỳnh Chi, 8A1, THCS Yên Phong, Yên Phong, Bắc Ninh; Nguyễn Thu Hiền, 8A3, THCS Thị trấn Kỳ Sơn, Kỳ Sơn, Hòa Bình.
BÙI MẠNH TÙNG Bài 4(183). Cho a, b, c là các số thực dương. Chứng minh rằng a2 b2 c2
1
ổ bỏ
+
+
b C a
√a2+b2 +√b2 +c2 +√c2+a2
Đẳng thức xảy ra khi nào?
Lời giải. Sử dụng bất đẳng thức AM-GM ta
Có
a2 + b2
2 2
b
+2b≥2√2(a2+b2) hay
a2 ≥ 2√2√a2 + b2 - 3b.
b
-
1
2
Do đó, với chú ý rằng vx + y 2 +
ta có
a2 b2 c2
√2
x2 y2 = = (x y)
+ ≥ 2√2(√a2 +b2 +
b с а
√b2 + c2 + √c2 + a2) – 3(a+b+c)
+
1
√ 1⁄2 (√a2 + b2 + √b2 + c2 + √c2 +a2
2
√2
3
√a2+b2 + √b2 + c2 + √c2 +
2+ a2
√2
-3(a+b+c)
{{(x+y)
(34
2
≥ 1/1 (√a2 + b2 + √b2 + c2 + √c2 +a2
+3(a+b+c) - 3(a+b+c)
2
= √1/2 (√a2 + b2 + √b2 + c2 + √c2 +a2
Đẳng thức xảy ra khi và chỉ khi a = b = c. Nhận xét. Các bạn sau có lời giải đúng cho bài toán: Nguyễn Thu Hiền, 8A3, THCS Thị Trấn Kỳ Sơn, Kỳ Sơn, Hòa Bình; Ngô Bảo An, Nguyễn Lê Tùng Dương, 9A0, THCS Nguyễn Trường Tộ, Đống Đa, Hà Nội; Trần Cao Bảo Châu, 7A, THCS Trần Hưng Đạo, Buôn Ma Thuật, Đắk Lắk; Trương Ngọc Tâm, 8D, THCS Nhữ Bá Sỹ, thị trấn Bút Sơn, Hoằng Hóa, Thanh Hóa; Huỳnh Nguyên Phúc, 9A1, THCS Mỹ Lộc, Phù Mỹ, Bình Định; Phùng Đăng Dương, 7C, THCS Văn Lang, TP. Việt Trì, Phú Thọ; Trần Phương Mai, 7B, THCS Hồ Xuân Hương, Quỳnh Lưu, Nghệ An.
Bài 5(183). Hai người chơi hai quân cờ S (sói) và T (thỏ) trên một bàn cờ như trong hình vẽ. Người A luôn dùng quân S, người B luôn dùng quân T. Mỗi bước đi được đẩy một quân cờ ở một đỉnh ô vuông của bàn cờ theo cạnh của ô vuông đến đỉnh ô vuông
CAO VĂN DŨNG
3
1
2
gần nhất. Bắt đầu là người A đẩy quân cờ S đi một bước, sau đó lần lượt đến người B đẩy quân cờ T một bước và cứ lần lượt như thế. Người A thắng nếu sau không quá 20 bước quân cờ S gặp được quân cờ T tại cùng một ô. Hỏi rằng A thắng hay không nếu:
a) S bắt đầu ở ô 1, còn T bắt đầu ở ô 2. b) S bắt đầu ở ô 1, còn T bắt đầu ở ô 3.
Lời giải. Trước hết, ta đánh số thứ tự các ô như hình vẽ.
a) Người A luôn thắng sau không nhiều hơn 7 nước đi của cả 2 người chơi A và B nếu A đi theo thuật toán sau:
- Nước đi thứ 1: Người A di chuyển quân cờ S từ ô số 1 sang ô số 12.
- Nước đi thứ 2: Người B di chuyển quân cờ T đến ô số 5 hoặc 11.
nước đi thứ 3 của
—
Nước đi thứ 3: Người A sẽ di chuyển quân cờ S đến vị trí ô số 3 sau khi người B di chuyển đến ô số 5, hoặc di chuyển S đến ô số 7 nếu B đã đi đến ô số 11
—
Nước đi thứ 4: Sau
4
5
2
3
6
11
12
7
10
các bạn đều giải sai. Có duy nhất bạn Hoàng Tuấn Dũng, 7A1, THCS Minh Khai, TP. Hà Giang, Hà Giang có lời giải tốt.
TRỊNH HOÀI DƯƠNG Bài 6(183). Cho tam giác ABC ngoại tiếp đường tròn tâm I. Gọi D là tiếp điểm của (I) và BC. Đường thẳng qua D vuông góc với AD cắt IB, IC theo thứ tự tại M và N. Chứng minh rằng tam giác AMN cân tại A.
Lời giải. Đường tròn (I) tiếp xúc với AB, AC tương ứng tại F, E.
Gọi K là giao điểm của BI và DE, L là giao điểm của CI và DF.
A
người A thì hai quân cờ S và T ở hai vị trí ô đối diện nhau của một ô vuông đơn vị nên ở nước đi thứ 4 này, người B đi đến các ô số 4, 6 (hoặc 6, 10) thì kiểu gì thì cũng bị người A chiến thắng ở nước đi thứ 5
1
8
9
vì quân cờ S đang ở ô 3 (hoặc ô số 5). Do đó người B buộc phải di chuyển quân cờ T về ô số 2 ban đầu.
- Nước đi thứ 5: Người A di chuyển quân cờ S về ô số 6.
Nước đi thứ 6: Người B di chuyển quân cờ T về ô số 5 hoặc 11.
- Nước đi thứ 7, người A di chuyển quân cờ S tới ô mà quân cờ T đang đứng ở đó và chiến thắng.
M
B
D
E
K
N
C
Giả sử L nằm trong đoạn DF và K nằm trong đoạn DE. Các trường hợp khác chứng minh
b) Người A không thể thắng nếu người B chơi với thuật toán
C
L
C
tương tự.
Dễ thấy
giữ vị trí quân cờ T cách 2 nước đi so với vị trí quân cờ S. Trên bàn cờ, ô có số chẵn
BAC ABC
ACB
L
C
L
AIK = IAB + IBA =
+
=
90°
2
2
2
=
= CED = 180° - AEK;
ghi là C, ô có số lẻ
C
L
C
BAC
ACB
ABC
ghi là L. Vì ban đầu
AIL = IAC +ICA =
= 90°
2
2
2
=
BFD = 180° - AFL.
C
L
L
quân cờ S và T tương ứng ở ô số 1 và số 3 có khoảng cách là 2 nước đi và có số ô cùng là số lẻ (L) nên người B chơi chiến thuật đi quân cờ T tới ô có số cùng tính chẵn lẻ với với vị trí quân cờ S ở nước đi ngay trước đó và không được tự đi vào ô mà quân cờ S đang đứng. Khi đó khoảng cách giữa hai quân cờ S và T sau nước đi của người B luôn là 2 nước đi nên người A không thể thắng (dù sau hơn 20 nước đi).
Nhận xét. Đây là bài toán khá thú vị, tương đối khó và là loại hình mới với học sinh THCS ở Việt Nam nên tuy có nhiều bạn gửi bài đến tòa soạn, nhưng rất tiếc là hầu hết
Do đó các tứ giác AEKI, AFLI nội tiếp. (1) Vậy AKM= AKI = AET = 90° = AFI = ALI = ALN. Kết hợp với AD I MN, suy ra các tứ giác AKDM, ALDN nội tiếp. (2)
Từ (1) và (2) suy ra
DAM=DKM = 180° - EKI-EAI=CAI=BAI
=FAI=180° - FLI= DLI=DLN = DAN.
Từ đó, chú ý rằng AD L MN, suy ra tam giác AMN cân tại A.
Nhận xét. Không có bạn nào giải đúng bài
NGUYỄN MINH HÀ
này.
35
Kết quả kì thi Olympic quốc tế của học sinh Việt nam năm 2018
• Kì thi Olympic Toán học quốc tế lần thứ 59 năm 2018 (IMO 2018) diễn ra tại Romania với sự tham dự của 615 thí sinh đến từ 110 quốc gia và vùng lãnh thổ. Kết quả cả 6 thí sinh của đoàn Việt Nam dự thi đều đoạt huy chương. Thí sinh Nguyễn Quang Bin, lớp 12, trường THPT chuyên Khoa học Tự nhiên, Đại học Quốc gia Hà Nội đoạt huy chương Vàng. Huy chương Bạc thuộc về 2 thí sinh: Phan Minh Đức, lớp 11, trường THPT chuyên Hà Nội-Amsterdam, Hà Nội; Trịnh Văn Hoàn, lớp 12, trường THPT chuyên Trần Phú, TP. Hải Phòng. Huy chương Đồng thuộc về các thí sinh Trần Việt Hoàng, lớp 12, trường THPT chuyên Trần Phú, TP. Hải Phòng; Trương Mạnh Tuấn, lớp 12, trường THPT chuyên Khoa học Tự nhiên, Đại học Quốc gia Hà Nội; Đỗ Hoàng Việt, lớp 12, trường THPT chuyên Nguyễn Quang Diệu, Đồng Tháp.
o Kì thi Olympic Vật lí quốc tế lần thứ 49 năm 2018 được tổ chức tại Cộng hòa Bồ Đào Nha từ ngày 21 đến 29/7/2018, với sự tham gia của 86 nước và vùng lãnh thổ. Việt Nam là một trong số 10 nước đoạt từ 2 huy chương Vàng trở lên và xếp thứ hạng cao. Cả 5 thí sinh của Việt Nam dự thi đều đoạt giải. Huy chương Vàng thuộc về các thí sinh Nguyễn Ngọc Long, lớp 12, trường THPT chuyên Lam Sơn, Thanh Hóa; Trần Đức Huy, lớp 12, trường THPT chuyên Hà Nội - Amsterdam, Hà Nội. Thí sinh Nguyễn Xuân Tân, lớp 11, trường THPT chuyên Khoa học Tự nhiên, Đại học Quốc gia Hà Nội và Trịnh Duy Hiếu, lớp 11, trường THPT chuyên Bắc Giang, Bắc Giang đoạt huy chương Bạc; Nguyễn Văn Thành Lợi, lớp 12, trường THPT chuyên Quang Trung, Bình Phước đoạt Huy chương Đồng.
Kì thi Olympic Hoá học quốc tế năm 2018 lần thứ 50 tổ chức tại Cộng hòa Séc và Cộng hòa Solovakia với sự tham dự của 304 thí sinh đến từ 82 quốc gia và vùng lãnh thổ. Cả 4 thí sinh của Việt Nam dự thi đều đoạt huy chương. Huy chương Vàng thuộc về thí sinh Phạm Đức Anh, lớp 12, trường THPT chuyên
36
Khoa học Tự nhiên, ĐHQG Hà Nội. Huy chương Bạc thuộc chương Bạc thuộc về Nguyễn Văn Chí Nguyên, lớp 11, trường THPT chuyên Lam Sơn, Thanh Hóa và Hoàng Thanh Tùng, lớp 12, trường THPT chuyên Lê Hồng Phong, Nam Định. Huy chương Đồng thuộc về Phan Nhật Duật, lớp 12, trường THPT chuyên Phan Bội Châu, Nghệ An.
• Kì thi Olympic Sinh học quốc tế năm 2018 lần thứ 29 tổ chức ở nước Cộng hòa Hồi giáo Iran, có 71 quốc gia và vùng lãnh thổ tham dự với tổng số 261 thí sinh. Kết quả cả 4 thí sinh dự thi đều đoạt huy chương. Huy chương Vàng thuộc về các thí sinh Nguyễn Phương Thảo, lớp 12, trường THPT chuyên Khoa học tự nhiên, Đại học Quốc gia Hà Nội; Trần Thị Minh Anh, lớp 12, trường THPT chuyên Trần Phú, Hải Phòng; Hoàng Minh Trung, lớp 11, trường THPT chuyên Lam Sơn, Thanh Hóa. Thí sinh Hoàng Văn Đông, lớp 12, trường THPT chuyên Nguyễn Trãi, Hải Dương đoạt Huy chương Bạc. Đặc biệt, thí sinh Nguyễn Phương Thảo đạt tổng điểm cao nhất cuộc thi trên tổng số 261 thí sinh.
• Kì thi Olympic Tin học Châu Á năm 2018 được tổ chức theo hình thức thi trực tuyến với 586 thí sinh thuộc 31 nước và vùng lãnh thổ tham gia, Cộng hòa Liên bang Nga là nước đăng cai. Việt Nam có 7/7 thí sinh tham gia xét giải đều đoạt giải. Huy chương Vàng thuộc về Phạm Đức Thắng, lớp 12, THPT chuyên Khoa học Tự nhiên, Đại học Quốc gia Hà Nội. Huy chương Bạc thuộc về các thí sinh: Hoàng Xuân Nhật, lớp 12, THPT Năng khiếu, Đại học Quốc gia TP. Hồ Chí Minh; Nguyễn Khánh, lớp 12; Nguyễn Minh Tùng, lớp 11, THPT chuyên Khoa học Tự nhiên, Đại học Quốc gia Hà Nội; Trịnh Hữu Gia Phúc, lớp 11, THPT chuyên Lam Sơn, Thanh Hóa. Huy chương Đồng thuộc về: Nguyễn Hoàng Hải Minh, lớp 12, THPT chuyên Khoa học Tự nhiên, Đại học Quốc gia Hà Nội và Dương Quốc Hưng, lớp 12, THPT chuyên Thăng Long, Lâm Đồng.
MAI VŨ
000000
000000
000000
GOBBED
Lịch sử Toán học
1. Mở đầu
TOÁN CHUYỂN ĐỘNG ĐỀU TRONG HAI CUỐN SÁCH TOÁN CỔ HÁN NÔM NƯỚC TA
TẠ DUY PHƯỢNG (Viện Toán học, Hà Nội) ĐOÀN THỊ LỆ (Đại học Thanh Hoa, Đài Loan) CUNG THỊ KIM THÀNH, PHAN THỊ ÁNH TUYẾT
Toán chuyển động đều hiện nay được giảng dạy trong chương trình lớp 5. Dạng toán chuyển động đều có lẽ đầu tiên được trình bày trong cuốn Cửu chương toán thuật của Lưu Huy (Lui Hui, Trung Hoa, thế kỉ III), là cuốn sách mà vai trò của nó trong phát triển toán học Trung Hoa được coi là tương đương với cuốn Cơ sở của Euclid ở phương Tây. Cửu chương toán thuật là một trong 10 cuốn sách toán được dạy trong Quốc tử giám Trung Quốc từ thời nhà Đường và cũng có ảnh
hưởng lớn đến phát triển toán học tại Nhật
Bản, Triều Tiên, Việt Nam.
Dạng toán chuyển động đều cũng đã được đề cập đến trong một số sách toán Hán Nôm nước ta. Bài viết này giới thiệu dạng toán chuyển động đều trong hai cuốn sách toán Hán Nôm Ý Trai toán pháp nhất đắc lục của Nguyễn Hữu Thận và Bút toán chỉ nam của Nguyễn Cẩn.
2. Các công thức cơ bản
Công thức cơ bản 1. (Chuyển động ngược chiều) Hai vật cách nhau quãng đường s khởi hành cùng lúc và chuyển động đều ngược chiều với vận tốc V1 và V2. Sau thời gian t chúng gặp nhau thì s = S + S2 = Vịt+V2t. Suy ra t=s: (V1 +V2).
1
(1)
Công thức cơ bản 2. (Chuyển động cùng chiều) Hai vật cách nhau một quãng đường độ dài s khởi hành cùng lúc và chuyển động đều cùng chiều với vận tốc V1 và V2 (V2 >V1). Sau thời gian t chúng gặp nhau. Ta có v2t = s +Vịt.
—
(2)
Suy ra t=s: (V2 − V1). Công thức cơ bản 3. Hai vật chuyển động
đều với vận tốc v1 và v2 thì tỉ lệ quãng đường đi được s1 và S2 bằng tỉ lệ vận tốc: S1_V1.
=
S2 V2
(3)
Ba công thức (1), (2), (3) là ba công thức cơ bản giải các bài toán chuyển động đều. Toán chuyển động đều trong Ý Trai toán pháp nhất đắc lục
Cuốn sách Ý Trai toán pháp nhất đắc lục (Một điều tâm đắc về toán của Ý Trai) của
Nguyễn Hữu Thận hoàn thành năm 1829 đã
được giới thiệu sơ bộ trong [3], [4]. Để dễ đọc, chúng tôi trình bày lời giải gần với ngôn ngữ hiện đại, nhưng vẫn cố gắng trung thành với lời giải trong các sách toán Hán Nôm. Toán chuyển động đều trong Ý Trai toán pháp nhất đắc lục [2] được Nguyễn Hữu Thận gọi là hành trình khoai mạn (hành trình nhanh chậm).
Bài toán 1. Giáp đi được 80 dặm một ngày, Ất đi 48 dặm một ngày. Ất đã đi trước được 480 dặm thì Giáp mới xuất phát. Hỏi Giáp đi bao nhiêu dặm và đi bao nhiêu ngày thì đuổi
Át?
kip
Lời giải. Thời gian Giáp và Ất cùng đi là: t=s:(V2-V1)=480 : (80 – 48) = 15 (ngày). Vậy quãng đường hai người đi cho tới khi gặp nhau là 15 × 80 = 1200 (dặm).
Bài toán 2. Người đi nhanh trong một ngày đi được 95 dặm, người đi chậm trong một ngày
đi được 75 dặm. Nay người đi chậm đã đi được 8 ngày. Hỏi người đi nhanh phải đi trong bao nhiêu ngày để có thể đuổi kịp người kia? Và đoạn đường dài bao nhiêu dặm? Lời giải. Quãng đường người đi chậm đi trong tám ngày là s = 75 × 8 = 600 (dặm).
37
Vậy thời gian để người đi nhanh đuổi kịp người đi chậm là:
t=s:(V2 - V1)=600 : (95 – 75)= 30 (ngày). Đoạn đường dài 95 × 30 = 2850 (dặm). Bài toán 3. Giả sử người đi chậm đã đi được 7 ngày, sau đó người đi nhanh đi theo được 6 ngày thì gặp người kia. Lộ trình của họ là 1170 dặm. Hỏi mỗi người mỗi ngày đi được quãng đường bao nhiêu?
Lời giải. Người đi nhanh đi với vận tốc 1170 : 6 = 195 (dặm/ngày).
Người đi chậm đi hết 7 + 6 = 13 (ngày). Vậy người đi chậm đi với vận tốc 1170 : 13 = 90 (dặm/ngày).
Bài toán 4. Giả sử có một người đi bộ, một người cưỡi ngựa. Người đi bộ đi trước 37 dặm, người cưỡi ngựa đuổi đến 154 dặm, còn 23 dặm mới đuổi kịp người đi bộ. Hỏi người cưỡi ngựa còn phải đi bao nhiêu dặm mới đuổi kịp người đi bộ?
Lời giải. Gọi vận tốc của người cưỡi ngựa là V2, vận tốc của người đi bộ là V1. Thời gian người cưỡi ngựa đi hết quãng đường 154 dặm cũng bằng thời gian người đi bộ đi hết quãng đường (154 – 37) + 23 = 140 (dặm). Vậy ta có
154 140
t
=
V2
=
=
Suy ra
V2 154
V1
V1 140
=
11
10
Gọi quãng đường người cưỡi ngựa còn phải đi cho tới khi gặp nhau là s. Theo bài ra ta có
11 V2 S
=
=
10 V1 S-23
Suy ra s = 253 dặm.
Để so sánh, chúng tôi đưa thêm một bài trong Bút toán chỉ nam ([1], in năm 1909), tuy ngôn ngữ của đề bài là mới, nhưng phương pháp giải vẫn như cũ (sử dụng ba công thức cơ bản trên).
Bút toán chỉ nam được Nguyễn Cẩn in năm 1909. Tuy Bút toán chỉ nam được viết bằng chữ Hán, nhưng Nguyễn Cẩn đã chịu ảnh hưởng của toán học phương Tây (do Pháp đưa vào Việt Nam cuối thế kỉ XIX). Thí dụ, ông đã trình bày phép toán cộng trừ nhân chia theo hàng dọc, nghĩa là khá gần với cách dạy toán hiện nay.
38
Bài toán 5. Chiều dài quãng đường từ trạm xe lửa Đông Khê đến Thái Nguyên là 52800 xích. Mỗi giờ xe ngựa đi được 9000 xích, xe tay đi được 7500 xích. Một xe đi từ Đông Khê
đầu đi từ 6 giờ thì mấy giờ sẽ gặp nhau và ở lên, một xe đi từ Thái Nguyên xuống đều bắt
nơi gặp nhau đó, mỗi xe đã đi được bao nhiêu xích?
Lời giải. Thời gian hai xe đi từ nơi xuất phát đến lúc gặp nhau là:
t=s: (V1+V2) = 52800: (9000 + 7500)
=
= 3,2 (giờ); 3,2 giờ = 3 giờ 12 phút. Hai xe gặp nhau lúc:
6 giờ + 3 giờ 12 phút = 9 giờ 12 phút. Xe ngựa đã đi: 9000 × 3,2 = 28800 (xích); Xe tay đã đi: 7500 × 3,2 = 24000 (xích). Kết luận. Chuyển động đều là dạng toán có ý nghĩa thực tế, đã được biết đến từ gần 2000 năm trước và cũng đã được phổ biến tại Việt Nam, ít nhất là cách đây khoảng 200 năm trong Ý Trai toán pháp nhất đắc lục. Bạn đọc có thể xem thí dụ, [5], để biết kĩ hơn về dạng toán chuyển động trong chương trình toán hiện nay.
Tài liệu trích dẫn
[1] Bù F Nguyễn Cẩn, 3 tôn Thi Bút toán chỉ nam, 1909, Thư viện Hán Nôm: A. 1031. [2] 阮有慎Nguyên Hou Thân, 意齋算法 1 Ek Ý Trai toán pháp nhất đắc lục (1829), Thư viện Hán Nôm: A.1336; VHv.1184; A. 982; A.1336/a.
[3] Tạ Duy Phượng, Đoàn Thị Lệ, Cung Thị Kim Thành, Phan Thị Ánh Tuyết, Đôi nét về nhà toán học Việt Nam đầu thế kỉ XIX, Ý Trai Nguyễn Hữu Thận và cuốn sách Ý Trai toán pháp nhất đắc lục của ông, Tạp chí Toán học và Tuổi trẻ, số 481 (tháng 7, 2017).
[4] Tạ Duy Phượng, Đoàn Thị Lệ, Cung Thị Kim Thành, Phan Thị Ánh Tuyết, Sai phân pháp trong Ý Trai toán pháp nhất đắc lục, Tạp chí Toán Tuổi thơ 2, số 183 và số 184 (tháng 4 và tháng 5-2018).
[5] Vũ Dương Thụy, Nguyễn Danh Ninh, Các bài toán số học về chuyển động đều, Nhà xuất bản Giáo dục, 2001 (Tái bản lần thứ nhất).
VƯỢT VŨ MÔN
B
BẤT ĐẲNG THỨC VÀ CỤC TRI ĐẠI SỐ QUA KÌ THI TUYỂN SINH VÀO LỚP 10 CHUYÊN TOÁN NĂM HỌC 2018-2019
ài viết này chúng tôi xin được giới thiệu cùng bạn đọc một số bài toán bất đẳng thức và cực trị đại số qua
kì
thi tuyển sinh vào lớp 10 chuyên toán năm học 2018 - 2019.
Bài toán 1. Cho ba số dương x, y, z thỏa
mãn x +y+z=1.
Chứng minh rằng
NGUYỄN ĐỨC TẤN (TP. Hồ Chí Minh)
Bài toán 2. Chứng minh rằng x^ −x+
với mọi số thực X.
2
1/1/1> 00
(Đề thi tuyển sinh vào 10 chuyên toán TP. Hồ Chí Minh)
Hướng dẫn giải. Ta có
1
x4.
-X +
2
=
4 2
1
2
X
+ X
x2 -
X+
4
4
2
2
1
+X
-
> 0.
2
2
ху √xy + z
yz
ZX
3
+
+
<
yz + x
zx + y
2
(Đề thi tuyển sinh vào 10 chuyên toán
= X
2
-
Đẳng thức xảy ra khi và chỉ khi
tỉnh Hà Tĩnh)
x2
2
đẳng thức
112
Hướng dẫn giải. Áp dụng bất AM-GM cho hai số dương, ta có:
VI
xy
xy + z
-12
X
=
2x+z
xy
xy + z(x + y + z)
1
0 và X = 0. Suy ra không có giá
2
trị nào của x để đẳng thức xảy ra.
1
X
y
=
Vậy x − x +
>0 với mọi số thực x.
2
x+z y + z
Bình luận. Bài toán với đánh giá chặt chẽ
hơn là: Chứng minh rằng x^-x+-2
y +
y + z
ху
1 X
y
1
<
+
. (1)
(4 –33/2) với x là số thực.
8
Vây
√xy + z
2x+z y + z
Chứng minh tương tự ta có
4
1
2
yz
1 y
Z
<
+
.(2)
2x + y
X+Z
1 Z
X
<
+
(3)
2
y + z
x + y
Vyz + x
ZX
zx + y
Từ (1), (2) và (3) ta có
xy
+
yz
+
ZX
3
<
2
xy + z Vyz+X
Vyz + x √zx + y
Bình luận. Đây là một bài toán quen thuộc, từ điều kiện rằng buộc x + y + Z = 1, có được xy+z = (X +z)(y + z) để đến lời giải.
Bài toán 3. Với x, y, z là các số thực dương
1 1 1
thay đổi và thỏa mãn -+-+-=3. Tìm giá
39
X
Z y
trị lớn nhất của biểu thức
P=-
1
+
1
1
+
√2x2 + y2+3 √2x2+z2 +3 √222+x2+3 (Đề thi tuyển sinh vào lớp 10 chuyên toán TP. Hà Nội) Hướng dẫn giải. Ta có 2(x – y) ≥ 0 nên 9xy ≤ 4xy + 2x + 2y + xy. Suy ra 9xy < (2x + y)(2y + x). Mà x, y > 0 nên ta có
1
2y + x
1
<
<
2 1 +
(1)
9xy
2x + y
9x 9y
2x + y
Từ (1) và áp dụng bất đẳng thức AM-GM cho
hai số dương, ta có
1
1
√2x2 + y2+3 √2(2x2 + 1) + y2 +1
1
√6
2
√6
1
<
<
4x+2y
4 √(2x+y).3
6 2 1 1
+
4 9x 9y 3
1
2x2 + y2+3 Tương tự ta có:
2y2 +z
+
1
4 2x+y 3
√62 1 1
+ +
4 9x 9y 3
(2)
1
6 2 1 1
+
+
(3)
2
+z2+3
4 9y 9z 3
<
1
√2z2 + x2+3
2
√6(2 1 1
+ + 4 9z 9x 3
Từ (2); (3) và (4) ta có
√6(2 1
1
P<
+
+
+
2
1
+ +
4 9x 9y 3 9y 9z
1 2 1 1
+ + +
—
3 9z 9x 3
=
√6
2
(4)
Đẳng thức xảy ra khi và chỉ khi x =y=z=1.
Vậy giá trị lớn nhất của P là
√6
2
2
x2
+
x2
y2
+4(y-2)+
X-2
y-2 X-2 y-2
+4(x-2)-4y -4x+16
2
X
≥2
y-2
y2
+2√
.4(x − 2) – 4y
X-2
.4(y-2)+2
-4x+16=4x+4y-4y - 4x + 16 = 16.
Bài toán 5. Các số thực x, y, z không âm thỏa mãn x + y + z + xy + yz + zx = 6. Tìm giá trị lớn nhất và giá trị nhỏ nhất của biểu thức Q =x+y+Z.
(Đề thi tuyển sinh vào lớp 10, THPT chuyên Đại học Sư phạm TP. Hồ Chí Minh) Hướng dẫn giải. Áp dụng bất đẳng thức AM- GM cho hai số không âm ta có
Q2 = (x + y + z)2 = x2 + y2 + z2 + 2xy + 2xz + 2yz ≤ x2 + y2 + z2 + x2y2 + 1 + x2z2 + 1 + y2z2 + 1=6+3=9.
Do đó Q < 3.
Đẳng thức xảy ra khi x = y = z = 1.
Vậy giá trị lớn nhất của Q = x+y+z là 3. Ta có 6 = x + y + z + xy + yz + zx 2x + y2 + x2y2 ≥ 2xy + x2y2 > xy + x2y2.
-
⇒x2y2+xy-6≤ 0⇒ (xy − 2)(xy + 3) ≤ 0 ⇒xy - 2 ≤ 0⇒ 2xy ≥ x2y2.
Tương tự ta có 2yz ≥yz, 2zx ≥ zx Do đó Q = x + y + z + 2xy+2yz + 2zx
≥ x2 + y2 + z2 + x2y2 + y2z2 + z2x2 = 6. Mà x, y, z ≥ 0 nên Q =x+y+z>0. Ta có Q = x+y+z2 Vô. √б.
Q
Bình luận. Việc nhận ra P đạt giá trị lớn nhất Đẳng thức xảy ra khi x = V6, y=0, z=0.
1
khi x = y = z = 1 kết hợp điều kiện ràng buộc
1 1 1
-+-+− = 3 giúp ta có được lời giải.
X y Z
:
Bài toán 4. Chứng minh rằng với x, y là các
số thực lớn hơn 2 thì
x2
≥ 16. y-2 x-2
+
(Đề thi tuyển sinh vào lớp 10, THPT chuyên Đại học Sư phạm TP. Hồ Chí Minh)
Hướng dẫn giải. Áp dụng bất đẳng thức AM-GM cho hai số dương, ta có
Bình luận. Dự đoán giá trị lớn nhất của Q là 3 khi x = y = z và giá trị nhỏ nhất của Q là 16 khi x = V6, y = z = 0, từ đó biến đổi có được lời giải.
Bài toán 6. Cho x, y, z là các số thực không âm và thỏa mãn x + y + z = 1. Tìm giá trị nhỏ nhất của biểu thức sau
T = (2x + 3y+4z)(5x + 2y + z).
(Đề thi tuyển sinh vào lớp 10 chuyên toán tỉnh Tây Ninh) Hướng dẫn giải. Vì x, y, z ≥ 0 và x +y+z=1 nên 9 – y ≤ 9.
40
Áp dụng bất đẳng thức AM-GM cho hai số không âm, ta có
7x-2y
Do đó
. (1)
√8x2 + 3y2+14xy
25
.4(4x+6y+8z)(5x + 2y + z)
1
T=
1
==
8
≤ — (4x+6y+8z+5x + 2y + z)2
= [9(x − 1
+ y + z) − y]2 = _—_(9 − y)2 ≤ — _.92
Đẳng thức xảy ra khi
4x+6y+ 8z = 5x + 2y + z
y = 0
7
y2
<
81
=
8
Tương tự ta có
√8y2+3z2 +14yz
z2
Z
8z2+3x2+14xz
Từ (1); (2) và (3) ta có
x2
8x2+3y2+14xy
7y-2z
(2)
25
7z-2x
>
(3)
25
+
y2 √8y2+3z2 + 14yz
x+y+z=1
Vậy giá trị lớn nhất của T là
81
8
X
-
8
y = 0
z2
2
+
1
Z=
8
Bình luận. Ta “chỉnh số thích hợp” để xuất hiện x = y = z và áp dụng bất đẳng thức AM-GM cho hai số không âm, rồi chọn điều kiện |9 – y| lớn nhất.
Bài toán 7. Cho x, y, z là các số thực dương. Chứng minh rằng
2
x2
X
√8x2 + 3y2+14xy
z2
+
√8z2
+3x2 + 14xz
+
y2 √8y2+3z2 +14yz
x + y + z
5
(Đề thi tuyển sinh vào lớp 10 chuyên toán
tỉnh Thái Nguyên)
Hướng dẫn giải. Áp dụng bất đẳng thức AM-GM cho hai số dương, ta có
x2 8x2+3y2+14xy+(x−y)2
x2
√8x2+3y2+14xy
x2
x2
x2
3x+2y 3x+2y
+
(3x+2y)2 3x+2y 3x+2y
25
25
ΔΙ
x2 3x+2y 3x+2y_7x-2y
>2
3x+2y 25
25
25
41
√8z2+3x2+14xz
>
x + y + z
5
Bài toán 8. Cho a, b là hai số nguyên thỏa mãn a + b >0.
a) Chứng minh rằng a + b > a+b>0. b) Chứng minh rằng a + b > a + b
(Đề thi tuyển sinh vào lớp 10 chuyên toán, trường PTNK, ĐHQG TP. Hồ Chí Minh) Hướng dẫn giải. a) Ta có a + b > 0
⇒ a3 > -b3⇒ a3> (-b)3 ⇒ a > -b⇒ a + b > 0.
2
Mà a2 – ab + b2
b
362
= a
+
≥ 0.
2
4
Đẳng thức xảy ra ea=b=0.
Do đó a + b = 0 (mâu thuẫn với a + b>0). Suy ra a – ab + b > 0.
Mà a, b = Z . Do đó a – ab + b >1.
Ta có (a + b)(a – ab + b) ≥ (a + b).1
3
→ a3 + b3 > a+b> 0.
b) Ta có a + b > 0 → a, b không đồng thời âm = ab < 0 hoặc a, b > 0.
• Nếu ab < 0 thì a? – ab + b > a + b.
+b3
Mà a + b ≥ 1 nên a + b = (a + b)(a? – ab + b) ≥ a2 – ab + b2 ≥ a2 + b2.
• Nếu a, b > 0 thì a > 1, b > 1 (vì a, b e Z). Do đó a > a, b > b.
3
3
Suy ra a + b > a+b.
3
3
Vậy a + b > a+b.
Bình luận. Đây là bài toán bất đẳng thức với các biến là các số nguyên, chú ý rằng với số nguyên x > 0 thì x ≥ 1.
VƯỢT VŨ MÔN
C
VÀI PHƯƠNG PHÁP GIẢI
PHƯƠNG TRÌNH VÔ TỈ
ThS. PHẠM MINH TÚ
(GV. Khoa CNTT, Trường Đại học Lao Động - Xã Hội, Hà Nội)
ác bài toán về phương trình vô tỉ thường xuất hiện trong các đề thi học sinh giỏi Lời giải. ĐKXĐ XS lớp 9 và đề thi vào lớp 10 THPT, bài viết này giới thiệu phương pháp đặt ẩn phụ kết hợp với phân tích các biểu thức để giải một số dạng toán về phương trình vô tỉ. Bài toán 1. Giải phương trình
2 42
√1-x2 + √ √ x2 + x − 1 + √1−x = 1.
X
-1+√5
Lời giải. ĐKXĐ
≤x≤1.
2
2
Đặt V1− x = a, 4x2 +x-1=b,
61-x
= c (0 ≤ a, b, c ≤ 1).
Ta có a+b+c=1;a + b + c = 1; b < 1, c <1.
a(1-a) ≥ 0
aza2
⇔
c≥c®.
Suy ra b(1-b3)>0b4
c(1-c5) ≥0
2 4
Do đó a+b+cza thể trô
b + C = 1.
Đẳng thức xảy ra khi và chỉ khi
a =
a =
0
1
b=0
b = 1
3-√5
2
3+ √5
hoặc x>
2
2
11
39
+ > 0.
4
16
Ta có 4x2 - 11x+10= 2x
Vi V2x2 – 6x +2 20 nên x−120 =x21.
3+√5
Kết hợp với ĐKXĐ ta có x > 2
Phương trình đã cho tương đương với (x-1)√2x2-6x+2(4x2 - 11x+10) = 0
→ (x − 1)√2x2 − 6x + 2 + (2x2 − 6x+2)
(6x2-17x+12)=0. (1)
-
Đặt V2x2 − 6x + 2 = a (a>0).
Phương trình (1) trở thành
a2 + a(x − 1) – (6x2 - 17x+12) = 0
↔ (a − 2x + 3)(a + 3x − 4) = 0.
• TH1. a = 2x − 3, ta có
√2x2-6x+2=2x-3.
2x-3>0
2x2 - 6x + 2 = (2x − 3)2
312
3
0
X≥
2
C = 1.
2x2-6x+7=0
2 x
(vô nghiệm).
=
Với a = 0 hoặc a = 1 thì x = ±1 hoặc x = 0. Thử lại chỉ có x = 1 thỏa mãn.
Vậy phương trình có nghiệm duy nhất x = 1. Bài toán 2. Giải phương trình
4x2-11x+10= (x − 1)√2x2-6x+2.
—
3
2
+
512
0
• TH2. a = 4 - 3x → √√2x2-6x + 2 = 4 − 3x.
4-3x≥0
2x2 − 6x + 2 = (4 − 3x)2
42
I
x<
+13
9
1 7 ( x = 2)2 + 17 = 0
7
(vô nghiệm).
7
Vậy phương trình đã cho vô nghiệm.
Bài toán 3. Giải phương trình
1
8x2-13x+7=1+
X
13x
33x2-2.
Lời giải. ĐKXĐ x=0.
Phương trình đã cho tương đương với
2
(2x − 1)3 — (x2 - x − 1) = (x + 1)√√3x2
—
—
X
―
||
12x2 -8x+7
3(2x- +
3 3
2
2, 17
-> 0.
4
4
Suy ra phương trình (5) vô nghiệm.
Thử lại ta có phương trình có tập nghiệm là
S =
{}
Bài toán 4. Giải phương trình
3
x2+2=2√x3 +1.
Lời giải. ĐKXĐ x>−1.
↔ (x2 + 2)2 = 4(x3 + 1) ↔ x2 + 4x2 + 4 = 4x3 +4
x2-4x3 +4x2=0↔ x2(x-2)2 =
3
= 0
+x=0 hoặc x= 2 (thỏa mãn ĐKXĐ).
= (2x−1)3 = (x2−x−1) + (x+1) 3x2 −2. (1) Vậy phương trình có tập nghiệm là S = {0; 2}.
→ = + 1)√√3x2
2
Đặt 2x − 1 = a; 33x2 -2 = b.
b3= 3x2 − 2 = (2x − 1)(x+ 1) +(x2 − x − 1). (2)
Từ (1) và (2) ta có hệ phương trình
a3 = x2
3 = x2-x-1+(x+1)b (3)
3 2
b3 = x2-x-1+ (x + 1)a. (4)
X
Trừ vế theo vế của (3) cho (4) ta được
(a - b)(a2 + b2 + ab + x + 1) = 0.
• TH1. a = b → 2x-1=33x2
3/3x2-2.
⇒ (2x − 1)3= 3x2 - 2.
↔ 8x3 - 15x2 + 6x + 1 = 0.
Bài toán 5. Giải phương trình
x+y+z+4=2√√x − 2 + 4√y − 3 + 6√z-5.
Lời giải. ĐKXĐ x>2; y>3;z>5. Phương trình đã cho tương đương với
(x−2−2√x−2+1)+(y-3-4√y-3+4)
+(z-5-6√z-5+9)=0.
↔(√x −2 − 1)2 + (√y−3 −2)2 + (√z−5 −3)2 = 0.
x-2-1=0
x=3
√y-3-2=0y=7
Z-5-3=0
(thỏa mãn ĐKXĐ).
Z = 14.
Vậy phương trình có nghiệm duy nhất
(x; y; z) = (3; 7; 14).
X
← (8x + 1)(x − 1)2= 0 ↔
8
x = 1.
Bài tập vận dụng
• TH2. a2 + b2 + ab + x + 1 = 0
a
+ (b + 2)2 + 3a2
a
Mà Ib+=
За2
2
2
4
·+x+1=0. (5)
3a2
+
+x+1
4
≥ 3a2 + x + 1 = 32x = 1)2 + x + 1
3(2x-1)2 +x+1=
4
4
+x+1
Bài 1. Giải các phương trình sau
1
1
4
a) x+x+ + X+ = 2;
4
2
2
b) x2 + √x2 + 2008 = 2008;
c) √4x2+5x+1-2√x2 − x + 1 = 9x − 3;
−
d) 2√x+1+6√9−x2 +6√(x + 1)(9 − x2) = 38+10x − 2x2 -x3.
X
3
43
NHÌN ĐA CUỐN GIỚI
BÀI TOÁN HAY VỀ PHẦN NGUYÊN CỦA MỘT SỐ
TRƯƠNG QUANG AN
(GV. THCS Nghĩa Thắng, Tư Nghĩa, Quảng Ngãi)
Sau đây chúng tôi xin giới thiệu bài toán hay có chứa phần nguyên của một số trong kì thi Olympic toán học của Canada năm 1999. Kí hiệu [x] là các số nguyên lớn nhất không vượt quá x. Bài toán. Giải phương trình 4x2 −40[x]+51=0. Lời giải. Cách 1. Đặt [x] = y (ye Z).
Khi đó ta có 40y = 4x +51>0. Lại có y≤x 0
⇒ye {2;6;7;8}.
[1,5≤ y ≤8,5
y <2,5
y>5,5
Cách 2. Đặt [x]=ne Z, {x} = a = [0;1) thì x = [x]+{x} = n+a.
Phương trình đã cho trở thành 4(n + α)2 − 40n+51=0
⇔
4a2 +8na + 4n2 - 40n+51 = 0. (2) Phương trình (2) có nghiệm khi
A' = 16n2 - 4(4n2 - 40n+51)
= 160n-204 = 16 10n-
51
4
⇒n≥2.
51
≥ 0
4
Do a20 nên với n≥2 thì (2) có nghiệm
α = -n+10n
Viosa 1 nên ta có
n-51
4
[x] = 2
2≤x≤3
-n+10n
——
51
≥ 0
29
10n-
-51
51 zn2
• TH1.
4
x2
2 29
29 X =
4
X = ±
2
51
51
4
2
-n+10n
<1
10n
< (n + 1)2
4
4
⚫ TH2.
⚫ TH3.
⚫ TH4.
2
X
=
S:
=
2 2
2
2
[x] = 6
189 →
x2
=
4
[x] = 7
x2
=
229 →
[x] = 8
4
269
4
Vậy phương trình có tập nghiệm là
√29 3√21 √229 √269
6
DINFIN
5
2
11
2
2
(2≤ n ≤8
8≤x≤9
269
269 → X =
X = ±
2
2
[n≤2 ne {2;6;7;8}.
n≥6
Từ đẳng thức x=[x]+{x}=n+a = 10n-
51
4
thay n bằng 4 giá trị trên ta tìm được các nghiệm của phương trình như ở cách 1.
44
Kết quả Cuộc thi giải toán dành cho nữ sinh (2 số 183)
Bài 31NS. Đặt n – 2016n + 1016067 = m (n = N) ⇒(n-1008)2-m2 = −3
(n−1008+m)(n−1008 - m) = −3.
Vim e N nên n−1008+m>n-1008 – m
n-1008+m=1
n-1008-m = -3
⇒n=1007
n-1008+ m = 3
hoặc
⇒n=1009.
n-1008-m-1
Thử lại đúng. Vậy n = 1007 hoặc n = 1009. Nhận xét. Các bạn có lời giải đúng: Hà Như Nguyệt, 7E, Nguyễn Thị Diệu Linh, 81, Vũ Huyền Trang, 8H, THCS Văn Lang, TP. Việt Trì, Phú Thọ; Phạm Khánh Huyền, 8B, THCS Hoàng Xuân Hãn, Đức Thọ, Hà Tĩnh; Bùi Hà Linh, 7D, THCS Đặng Thai Mai, TP. Vinh; Trần Thiên Ngân, 7A, THCS Cao Xuân Huy, Diễn Châu, Nghệ An; Nguyễn Thu Hiền, 8A3, THCS Thị trấn Kỳ Sơn, Kỳ Sơn, Hòa Bình.
Bài 32NS. ĐKXĐ x>
1
4√3
Phương trình đã cho tương đương với
4
1+x2 = 4√3.√√4√3x – 1.
Đặt y = 14V3x−1 (y20) ta được
4
x2 = 4√√3y-1 Ly1 = 4√3x-1
1
(1), với x;y >
1
Với x>y>
4√3
x4 = 4√3x-1
2
→ x2 + 4x2 + 4 = 4x2+4√3x+3
2
→ (x2 + 2)2 = (2x+√3)2
1
ex+2=2x+V3 (vì x > )
4√3
(TTT2
+x=1+ VV3 −1 (thỏa mãn điều kiện).
Vậy tập nghiệm của phương trình đã cho là
S={1±√√√√3−1}.
Nhận xét. Không có bạn nào có lời giải đúng.
S = {{1 ± √ √3 = 1}
Bài 33 NS. Vì có 21 điểm được tô bởi 4 màu nên theo nguyên lí Dirichlet có ít nhất 6 điểm cùng màu. Giả sử các điểm đó là A, B, C, D, E, F.
,
Xét 5 đoạn thẳng AB, AC, AD, AE, AF được tô bởi 2 màu nên theo nguyên lí Dirichlet có ít nhất 3 đoạn thẳng cùng màu, giả sử AB, AC, AD cùng màu tím.
Để giải tiếp ta cần 2 Nhận xét dưới đây để tìm tam giác có ba cạnh cùng màu.
Nhận xét 1. Nếu hai đoạn thẳng MN và NP
thì x4 >y4, nên từ hệ (1) cùng màu thì chỉ cần xét đoạn thẳng MP với
suy ra y > x (mâu thuẫn với x > y).
1
Tương tự với y>x> cũng dẫn đến mâu
thuẫn. Suy ra x=y>
Do đó từ hệ (1) suy ra
4√3
1
4√3
45
màu khác (vì nếu MP cùng màu thì AMNP có 3 cạnh cùng màu).
Nhận xét 2. Nếu các đoạn thẳng MN, MP cùng màu và QN, QP cùng màu (nhưng khác màu với MN, MP) và đoạn NP có một trong hai màu trên thì luôn tồn tại tam giác có ba cạnh cùng màu.
• Trường hợp I. Nếu trong 3 đoạn thẳng BC, CD, DB có một đoạn thẳng màu tím, chẳng hạn BC thì khi đó tam giác ABC có ba cạnh cùng màu tím.
Từ Nhận xét 1 chỉ xét DB và DC màu đen. Xét hai trường hợp sau với DE.
I.a. Nếu DE màu đen, theo Nhận xét 1 chỉ xét CE màu tím, lúc đó CB, CE và DB, DE thỏa mãn Nhận xét 2 (Hình 1).
I.b. Nếu DE màu tím, theo Nhận xét 1 xét EA màu đen. Xét hai trường hợp sau đối với AF (Hình 2).
I.b.1. Nếu AF màu đen, theo Nhận xét 1 xét EF màu tím, lại xét DF màu đen, rồi xét CF màu tím. Lúc đó CB, CF và DB, DF thỏa mãn Nhận xét 2.
I.b.2. Nếu AF màu tím, theo Bổ đề 1 xét DF màu đen. Lúc đó AB, AF và DB, DF thỏa mãn Nhận xét 2.
B
C
A
D
Hình 1
A
F
F
B
E C
D
Hình 2
đen, suy ra ED phải màu tím. Lúc đó BD, BF và ED, EF thỏa mãn Nhận xét 2.
II.a.2. Nếu AE màu tím, theo Nhận xét 1 xét EB, EC màu đen. Lúc đó ABCE có 3 cạnh màu đen.
II.b. Nếu AF màu tím, theo Nhận xét 1 xét CF màu đen. Lúc đó AB, AF và CB, CF thỏa mãn Nhận xét 2 (Hình 4).
B
A
C
D
Hình 3
Il.a.1
F
B
A
F
E C
D
Hình 4 Il.b
E
Vậy với cách tô màu tùy ý luôn có hai tam giác có ba đỉnh cùng màu và ba cạnh cùng
màu.
Nhận xét. Bạn Hà Như Nguyệt, 7E, THCS Văn Lang, Việt Trì, Phú Thọ lập luận đúng luôn chọn được một tam giác có ba đỉnh cùng màu và ba cạnh cùng màu.
E
HH
• Trường hợp II. Nếu cả 3 đoạn thẳng BC, CD, DB cùng màu đen, khi đó tam giác BCD có ba cạnh cùng màu đen.
Xét hai trường hợp sau với AF.
II.a. Nếu AF màu đen.
Xét hai trường hợp sau đối với AE.
II.a.1. Nếu AE màu đen, theo Nhận xét 1 xét EF màu tím (Hình 3).
Xét hai trường hợp sau đối với BE.
Nếu BE màu đen, theo Nhận xét 1 xét EC, ED màu tím, rồi xét CF màu đen. Lúc đó CD, CF và ED, EF thỏa mãn Nhận xét 2.
•
Nếu BE màu tím, theo Nhận xét 1 xét BF
màu đen, lại xét CF màu tím, rồi xét CE màu
SINCE 1959
C CỤ VĂN PHÒNG PHẨM
HONG HA
lưu truyền thống - Thiết tương lại
Chỉ có duy nhất một bạn được
thưởng, đó là: Hà Như Nguyệt, 7E, THCS Văn Lang, Việt Trì, Phú Thọ.
NGUYỄN HIỆP
TIN TỨC - HOẠT ĐỘNG - GẶP GỠ
Ngày 16.9.2018, Cuộc thi vô địch Toán cấp Trung học Úc mở rộng (AIMO) năm 2018 đã được tổ chức tại trường THCS Cầu Giấy, Q. Cầu Giấy, Hà Nội. Hàng năm cuộc thi có các thí sinh của hơn 20 quốc gia và vùng lãnh thổ tham gia tranh tài. Năm nay có 940 thí sinh đến từ 50 trường THCS và THPT trên địa bàn thành phố Hà Nội tham dự, đây là lần thứ ba cuộc thi AIMO được tổ chức tại Việt Nam.
TTT
46
SAI Ở ĐÂU? SỬA CHO ĐÚNG
LỜI GIẢI ĐÚNG CHƯA?
ThS. ĐINH VĂN THƯ
(GV. CĐSP Thái Bình, TP. Thái Bình, Thái Bình)
Bài toán. Cho đường tròn tâm O, bán kính R và điểm P nằm bên trong đường tròn đó (P không trùng O). Gọi d là đường thẳng đi qua P cắt đường tròn tại hai điểm phân biệt A, B (d không qua đi tâm O). Tìm diện tích lớn nhất của tam giác AOB khi đường thẳng d di động nhưng luôn đi qua P.
B
P
H
A
d
⇒ OH =R2
OHR
AB2
4
Vậy diện tích của tam giác AOB là
1
SAOBOHAB = AB, R2
2
AB2
4
Theo bất đẳng thức Cauchy ta có
SAOB
=
1
-
AB., R2 2
1
AB2
2
4
AB2 AB
+R2 AB2
R2
AB2
4
2
4
=
2
O
Lời giải. Gọi H là hình chiếu vuông góc của điểm O lên đường thẳng d. Ta có H là trung điểm của đoạn thẳng AB, suy ra AH
HB:
AB
2
Xét tam giác vuông AHO có OH = OA – AH2
2
=
OA2
AB
2
=R2
AB2
4
Kết quả
(TTT2 số 183)
=
LỜI GIẢI NHƯ THẾ ĐÃ ĐÚNG CHƯA? Cho x, y, z là các số thực dương và x + y + z
<
5
2
Sử dụng phương pháp biến đổi tương
đương ta có bất đẳng thức
√a2 + b2 + √c2 + d2 ≥ √(a+c)2 + (b + d)2 (1) với a, b, c, d là các số thực dương.
Đẳng thức xảy ra ở (1) khi và chỉ khi ad = bc. Áp dụng bất đẳng thức (1) ta được
A =
=
2
2
1
1
x2 +
+ y2+ + Z
2
1
+
z2
2
.
Suy ra SAOB <
4
R2
2
Đẳng thức xảy ra khi và chỉ khi
AB
2
=
R2
AB2 AB2
=
R2 AB2
4
4
4
R2
2
→ AB = R√2.
Suy ra diện tích lớn nhất của AAOB là
khi tam giác AOB vuông tại O.
Theo bạn, lời giải trên đã hợp lí chưa? Tòa soạn đợi thư của bạn.
1
1 1
2
≥1(x + y + z)2 + + + (2)
X y Z
·
Đẳng thức xảy ra ở (2) khi và chỉ khi x = y = z. Biến đổi đến đây là đúng.
Áp dụng bất đẳng thức AM-GM ở về phải của (2) và sử dụng bất đẳng thức
(x + y + z)
1 1 1
+
-
-
+
X y z
ta được (x+y+z) +
(x+y+
ΔΙ
≥ 2(x + y + z)
>9 (3)
2
1 1 +
Z
+ +
1
X
1
1
1
+ + X y
Z
-
y
≥ √√2.9 = 3√2. (4)
47
Đẳng thức ở (4) xảy ra khi và chỉ khi
1
-
1
x + y + z = + +
X y
1
Z
Kết luận A ≥ 3 V2 sẽ đúng nếu hai lần biến
đổi ở (2) và (4) các đẳng thức đều xảy ra, tức
là x=y=z và 3x=
3
dẫn đến x=y=z=1,
X
"
nhưng lúc đó x+y+z=3>
5
=3>~, không thỏa mãn
2
đề bài. Như vậy biến đổi ở (4) đúng và A > 3V2, nhưng không tồn tại các giá trị x, y, z để A = 3 V2.
Chú ý rằng bất đẳng thức (3) luôn đúng với các số dương x, y, z tùy ý (bạn đọc tự chứng minh bất đẳng thức (3) khi khai triển vế trái và áp dụng bất đẳng thức AM-GM cho từng cặp số hạng), do đó ta vẫn có thể áp dụng bất đẳng thức AM-GM ở vế phải của (2) nhưng cần thêm hệ số m nào đó vào thừa số dạng phân số ở vế trái của (4).
Đặt x+y+z =as
1 1
+
-
1
> +
-
07
5
thì
2
9
=
X y Z x+y+Z
9
a
Từ (2) ta biến đổi tiếp như sau
B = (x + y + z)2 +
+ +
(x+y+z)2+m2
1 1 1
+
—
+
X y Z
1 1 1
+
-
2
2
=
+ +(1-m2)
x y Z
1
1 1
>2m(x + y + z) |
X
·+-+-+ (1-m2) (1−m2)
≥ 18m+ (1-m2) 81
2
a2
y Z
(5)
3
nên có 3x = m.
-
X
hay là m = x2
Thay giá trị của m vào (5) ta được
B≥ 18m+(1-m2)
81
a2
a 81
18-
9
+1
2 81
=
+
2
81 a2 V
a2
9
Đặt t = a ta cần tìm giá trị nhỏ nhất của hàm
81
t
25
số f(t)= t + với 0 0
t1t2
(vì 0
25 324
+
4 25
25
lúc đó
"
4
=
/1921
10
Từ đó và (2) ta có A > B >
√1921
10
1 1 1
Giá trị của A
√1921
=
đạt được khi x+y+z
-
+ +
10
5
2
= a=
-
2
3
33
5
=
6
1921
10
x y Z
1
-
1 1 + +
-
X y Z
Đẳng thức xảy ra ở (5) khi và chỉ khi x + y + z
và trong lần biển đổi thứ
1 1
= m
+
+
X
y
14
a
nhất thì đẳng thức xảy ra khi x = y = z =
3
48
và x=y=Z=
Vậy giá trị nhỏ nhất của A là
HH
SINCE 1959
PHÒNG PHẨM
B
HỒNG HÀ . Nhận xét. Đây là bài khó, chỉ có
lưu truyền thống - tiết tương lai
bạn Vũ Huyền Trang, 8H, THCS
Văn Lang, TP. Việt Trì, Phú Thọ đã phân tích đúng là các bất đẳng thức ở (2) và ở (4) không đồng thời xảy ra.
ANH KÍNH LÚP
Chữ (Kì này KÌ 34
Hãy thay các chữ cái bởi các chữ số. Các chữ khác nhau biểu diễn các chữ số khác nhau. Lời giải cần có lập luận lôgic.
DOS
X
DOS
CUATRO
ĐÔNG BA
Kết quả
X
KÌ 32 (TTT2 số 183)
TRO
HOPP
KARLEK
a) Nếu (P; C) bằng (2; 7) hoặc (7; 2), cho R các giá trị 1, 3, 5, 6, 8, 9 rồi xét E, H, T thấy không thỏa mãn.
b) Nếu (P; O) bằng (3; 8) hoặc (8; 3), cho R các giá trị 1, 2, 5, 6, 7, 9 rồi xét E, H, T thấy không thỏa mãn.
Tích của số có 3 chữ số với số có 4 chữ số thì c) Nếu (P; O) bằng (6; 9) hoặc (9; 6), cho R
có 6 hoặc 7 chữ số, do đó T.H 2.
Xét các trường hợp sau:
1) K = 3 thì (T; H) bằng (1; 2) hoặc (2; 1) và P.O = 3 + 10X với X là số nguyên. Từ đó (P; O) bằng (7; 9) hoặc (9; 7).
=
a) Nếu P = 7, O = 9 thì 77(10R + 9) = 770R + 693 nên 7R + 9 = E + 10X. Cho R các giá trị 4, 5, 6, 8 thì được giá trị tương ứng E là 7 (loại), 4, 1 (loại), 5. Thử thấy 1977.259 512043 (loại); 2977.159 473343 (loại); 1977.289 = 571353 (loại); 2977.189 = 562653 (loại).
=
b) Nếu P = 9, O = 7 thì 99(10R + 7) = 990R + 693 nên 9R + 9 = E + 10X. Cho R các giá trị 4, 5, 6, 8 thì được giá trị tương ứng E là 5, 4, 3 (loại), 1 (loại). Thử thấy không thỏa mãn.
3) K = 5 thì P.O = 5 + 10X nên một trong các chữ số P hoặc O bằng 5 (loại).
4) K = 7 thì P.O = 7 + 10X nên (P; O) bằng (3; 9) hoặc (9; 3). Cho R các giá trị 1, 2, 4, 5, 6, 8 rồi xét E, H, T thấy không thỏa mãn.
5) K = 9 thì P.O = 9 + 10X thì không có các chữ số P, O thỏa mãn.
:
6) K = 4 thì P.O = 4 +10X.
các giá trị 1, 2, 3, 5, 7, 8 rồi xét E, H, T thấy không thỏa mãn.
4) K = 6 thì P.O = 6 +10X.
a) Nếu (P; O) bằng (2; 8) hoặc (8; 2), xét tương tự như trên, cho R các giá trị 1, 3, 4, 5, 7, 9 được một nghiệm là 472.1288 = 607936. b) Nếu (P; O) bằng (4; 9) hoặc (9; 4), cho R các giá trị 1, 2, 3, 5, 7, 8 thấy không thỏa mãn.
c) Nếu (P; O) bằng (7; 8) hoặc (8; 7), cho R các giá trị 1, 2, 3, 4, 5, 9 thấy không thỏa mãn.
5) K = 8 thì P.O = 8 +10X.
a) Nếu (P; O) bằng (2; 9) hoặc (9; 2), cho R các giá trị 1, 3, 4, 5, 6, 7 thấy không thỏa mãn.
c) Nếu (P; O) bằng (3; 6) hoặc (6; 3). Cho R các giá trị 1, 2, 4, 5, 7, 9 thấy không thỏa mãn.
b) Nếu (P; O) bằng (4; 7) hoặc (7; 4). Cho R các giá trị 1, 2, 3, 5, 6, 9 thấy không thỏa mãn.
Bài toán chỉ có một nghiệm là
472.1288 607936.
Nhận xét. Rất tiếc không có bạn nào giải đúng kì này.
AN MINH
49
Phá án cũng tham từ Số Lộc Bốc
VỤ ÁN TRƯỚC CỬA HÀNG SÁCH
uổi sáng, trời thu trong xanh, từng làn gió mát lành thổi nhè nhẹ. Thám tử
\ DSê Lốc Cốc ngồi bên li cà phê với
dáng vẻ thật thảnh thơi, thanh nhàn. Nhìn đám trẻ con tíu tít ra vào cửa hàng sách phía đối diện, thám tử ước ao: “Giá như mình được trở lại tuổi học trò, chắc bây giờ mình đang được mẹ đưa vào hiệu sách kia, mình sẽ mua vài tập truyện trinh thám li kì, hấp dẫn...”.
Đang mơ màng nghĩ về thời quá khứ, thám tử chợt giật mình bởi tiếng cãi cọ:
- Ồ, sao chốn văn minh như hiệu sách mà lại xảy ra tiếng cãi cọ ồn ào như chợ búa thế nhỉ? Biết mua sách, đọc sách thì chắc chắn là người văn minh rồi. Sao mà lại om sòm thế chú nhỉ? - Cô bán cà phê thắc mắc. - Ừ, thôi cô cho tôi trả tiền cà phê, để tôi thử sang bên ấy xem sao.
- Ôi, chú cứ ngồi đây, tiện thể trông hàng cho cháu.
-
Cô gái mồm nói chân chạy, thám tử đành ngồi chờ. Một lát, cô gái tíu tít chạy về:
- Có một bà để túi tiền trong tủ gửi đồ chú ạ. Vào hiệu sách xong ra bị mất tiền. Lạ thật, đã quy định không được để đồ quý trong tủ gửi đồ rồi giờ mất lại đòi bắt đền.
- Bà ta mất nhiều tiền không?
- Hình như mấy chục triệu chú ạ!
- Thế bà ta có nghi ngờ ai không?
—
—
LÊ HỒNG MAI (Hà Nội)
Bà khách mất tiền vẫn đang đứng giữa đám đông tay xỉa xói về phía anh thợ sửa khóa:
Anh không trả tiền cho tôi thì không yên đâu. Chỗ làm ăn của người ta, anh ngồi đây làm ăn mà dám ăn trộm hả?
- Bà này kì lạ quá! Tôi không lấy tiền của bà. Tôi mở tủ bằng cách nào hả?
Anh là thợ sửa khóa. Khóa nào anh không mở được hả?
- Nhưng tôi không lấy của bà.
—
-
Anh không lấy, thế anh ngồi ngay đây, anh sẽ nhìn thấy người lấy.
- Lúc nãy tôi có khách gọi đi sửa khóa, tôi vừa về đến đây nên tôi không biết.
Ai làm chứng? Gọi người khách của anh đến đây!
Anh ta sửa khóa xe máy nên đi rồi. Mà tôi không việc gì phải thanh minh với bà. Thấy hai bên tranh cãi gay gắt, thám tử liền lên tiếng:
- Chị ơi, tôi là thám tử, tôi sẽ giúp chị tìm ra người lấy tiền của chị.
-
Tôi không cần thám tử, tôi biết chắc kẻ trộm là anh ta! - Bà mất tiền vẫn lu loa.
-
Thì phải có chứng cớ mới bắt người ta trả tiền được chứ. Sao chị chỉ nghi ngờ anh thợ khóa mà không nghi ngờ ai khác, ví dụ anh bảo vệ cửa hàng chẳng hạn.
- Bảo vệ cửa hàng không lấy vì nó là cháu tôi... - Bà ta đang nói bỗng dừng lại - Với lại
- Dạ, đang đổ riết cho anh thợ sửa khóa ngồi khi chọn sách tôi có để ý, tôi không thấy nó
gần đó.
- Cửa hàng có người bảo vệ không?
-
- Có chú ạ. Nhưng bà ấy chỉ nghi ngờ anh
thợ sửa khóa vì anh ta ngồi ngay gần tủ để đồ. - Ồ, vụ này thú vị đây.
Thám tử Sê Lốc Cốc đi sang cửa hàng sách.
50
rời khỏi chỗ cửa.
- Chị vào cửa hàng sách lâu chưa? Sao đi
—
mua sách mà chị mang nhiều tiền vậy?
Tôi vào được khoảng 1 tiếng, từ lúc cửa hàng vừa mở. Tôi vừa ra ngân hàng rút tiền về tiện thể ghé đây mua sách.
Thám tử liếc nhìn đồng hồ: 7 giờ 45 phút. - Bà mua sách gì vậy?
- Tôi mua Từ điển Tiếng Anh cho cháu tôi! Thám tử nghe xong đi vào cửa hàng sách. Ông đi sâu vào phía có giá để loại sách Từ điển. Đó là góc trong cùng của cửa hàng, nơi kín đáo nhất mà người ở cửa không thể nhìn thấy. Ông cầm quyển từ điển đi ra, qua quầy ông hỏi cô bán hàng:
- Cửa hàng mình mấy giờ mở cửa cô nhỉ? - Dạ, 7 giờ đúng ạ.
Thám tử đi về phía vị khách đáo để:
Tôi nghĩ chị nhầm lẫn gì chăng! Tôi đã tìm ra sự thật câu chuyện mất tiền này.
- Anh biết gì mà lại nói thế! - Bà mất tiền vẫn to tiếng.
- Để tôi chỉ cho chị ba chứng cứ dẫn đến kết luận của tôi nhé! Chị nên xin lỗi anh thợ khóa đi! Chị muốn tôi nói riêng hay tôi nói luôn trước mặt mọi người!
- Đồ dở hơi! Đang đâu chỗ mũi vào chuyện người khác - Bà ta nói xong rồi chuồn thẳng! Các thám tử Tuổi Hồng có tìm ra chứng cứ để minh oan cho anh thợ sửa khóa không?
0
6
0
Kết quả
(TTT2 số 183)
Vụ trộm trước
chuyến du lịch
BAMAN
Thám tử Sê Lốc Cốc nghi ngờ cậu Dũng, cháu bà Hiền bởi hai lí do: Thứ nhất, tên bộ phim phải là Batman (Người dơi) chứ không phải Badman. Thứ hai, bộ phim Batman đó của hãng DC Comics chứ không phải của hãng Marvel. Rất vui vì tất cả các bạn tham gia kì này đều tìm đúng sơ hở của cậu Dũng. Thám tử gửi lời cảm ơn tới tất cả các bạn.
HH
SINCE 1959
CTCP VĂN PHÒNG PHẨM
| HỒNG HÀ
chưa truyền thống - Diết tương lai
Phần thưởng kì này sẽ được gửi tới:
Vũ Minh Nguyệt, 6E, THCS Vĩnh Tường, Vĩnh Tường, Vĩnh Phúc; Nguyễn Thu Hiền, 8A3, THCS Thị trấn Kỳ Sơn, Kỳ Sơn, Hòa Bình; Ngô Xuân Nguyên, 7A1, THCS Yên Phong, Yên Phong, Bắc Ninh; Vũ Huyền Trang, 8H, THCS Văn Lang, Việt Trì, Phú Thọ; Lê Hoàng Minh, 7D, THCS Đặng Thai Mai, TP. Vinh, Nghệ An.
Thám tử Sê Lốc Cốc
51
GIẢI TOÁN THẾ NÀO?
MỘT SỐ DẠNG TOÁN VỀ TỈ LỆ THỨC
HÀ VĂN NHÂN (GV. THCS Hoằng Xuân, Hoằng Hóa, Thanh Hóa)
Trong chương trình lớp 7, dạng toán về tỉ lệ thức là một trong những kiến thức cơ bản và quan trọng. Tuy nhiên, có nhiều học sinh còn khá lúng túng khi gặp dạng toán này. Bài viết này giới thiệu phương pháp giải một số bài toán về tỉ lệ thức.
Tính chất của tỉ lệ thức và tính chất dãy tỉ số bằng nhau
* Tính chất 1. Nếu
a
b
=
C
thì ad = bc.
d
* Tính chất 2. Nếu ad = bc với a, b, c, d khác
0 thì ta có các tỉ lệ thức
a C a b d C d
=
b
"
=
"
=
d с d b a C
b
=
* Tính chất 3. Từ tỉ lệ thức
tỉ số bằng nhau
=
b d
* Tính chất 4. Từ
a
bằng nhau
a C
-
=
a
a
b
=
||
d
, suy ra dãy
а-с
b-d
Lời giải. Ta có
a
b
=
C
d
Ta lại có
b a
=
a-b
a4
cdc-d
a
c
=
b
d
介
Từ (1) và (2) suy ra
a
||
d4
a-b
4
=
C4 c-d
4 a
+b4
d4
·
4 4
a +b
(2)
||
4 C
4
a b
=
4
C +d4
c-d
(1)
Bài toán 2. Cho các số thực a, b, c, d thỏa
mãn a + c = 2b và 2bd =
+
c(b + d) (với b; d
a C
khác 0). Chứng minh rằng b d
Lời giải. Ta có a+c=2b=(a+c)d = 2bd. Kết hợp với giả thiết ta có
c(b + d) = (a + c)d⇒ cb + cd = ad + cd
⇒ad = bc⇒
a
b
C
-=
d
Bài toán 3. Cho các số thực a, b, c, d khác 0 và khác nhau thỏa mãn b = ac; c = bd và
-, suy ra dãy tỉ số và khác nhau thỏa mãn bả
a C
a+c
=
b+d
e
d
g
e
b+d+g
b
b d g
a+c+e a-c+e
b-d+g
* Tính chất 5. Cho a, b, c tỉ lệ thuận với x, y, z
tức là ta có
a b C
-= —=—
X Y Z
* Tính chất 6. Cho a, b, c tỉ lệ nghịch với x, y, z tức là ta có ax = by = cZ.
Trong các đẳng thức trên điều kiện là các mẫu khác 0.
Sau đây là một số bài tập vận dụng
3
b° + c +d = 0. Chứng minh rằng
3
a3+b3+c3
3
3
3
a
b3 +c3 +d3 b+c
=
||
ه الله
=
||
||
ejo
Lời giải. Ta có b = ac
a
b
(1)
b C
b
Ta lại có c? =bd=
(2)
C
Từ (1) và (2) suy ra
3
a b C
a
=
b
c d
3
3
a C
Bài toán 1. Cho
Chứng minh rằng Mặt khác
a
c
=
||
介
=
b
d
b c
d b3
4
a-b
a4+b4
==
c-d
c4+d4
52
Từ (3) và (4) suy ra
a3 +b3 b3+c3 +
=
σίο
b
b
3
d
3
3 a+b+c
3 b+c
+
3
a b c
b c d
3
+b+c
a
=
a
d
3
d3
(4)
(3)
Bài toán 4. Cho a, b, c là các số khác nhau và khác 0 thỏa mãn a(y + z)=b(z + x) =c(x + y). Chứng minh rằng
x + y
5-z
y + z
9+Y
k =
3
1
2
5
(x + y) + (5 − z) + (y + z) − (9 + y) _ x+y-4
y-z
=
Z-X
x-y
a(b-c) b(c-a) c(a - b)
Lời giải. Vì a; b; c khác 0 nên từ giả thiết và áp dụng tính chất dãy tỉ số bằng nhau ta có
a(y + z)
abc
y + z
bc
Y+z
=
=
=
bc
b(z+x) c(x + y)
abc
=
abc
Z+X
x + y
ac
ab
(x + y) − (z+x)
ab - ac
(y + z)-(x + y) __ (z+x)-(y + z)
bc-ab
y-z
Z-X
=
ac-bc
x-y
3+1+2-5
1
x + y − 4 = k
⇒k + 4 = x + y = 3k
x + y = 3k
⇒ 4+k=3k4= 2k → k = 2.
Do đó ta có
5-z=k⇒z=5-k=5-2 = 3;
9+y=5ky=5k-910-9=1;
x + y = 3k⇒x=3k-y=6-1=5. Vậy x =5, y =1, z=3.
Bài toán 7. Tổng các luỹ thừa bậc ba của ba số là –1009. Biết tỉ số giữa số thứ nhất và số
2
thứ hai là
a(b-c) b(c-a) c(a - b) `
3
; tỉ số giữa số thứ nhất và số thứ
4
Bài toán 5. Cho các số thực a, b, c, x, y, z với
ba là
Tìm ba số đó?
9
a, b, c khác 0 thỏa mãn
ay - bx
bz-cy
CX - az
a
b
Lời giải. Gọi ba số cần tìm là x, y, z.
X
Chứng minh rằng
y
-
=
Ta có x
3 3 3
+ y + z = –1009;
X
==
C
a
b
y
213
↑
4
||
X
=
9 4
9
IN
X
y
介
4
||
NO
||
16
||
Z
2
X | N
=
y
3
=k
9
=
Z
C
Lời giải. Từ giả thiết và áp dụng tính chất dãy
tỉ số bằng nhau ta có
cx - az ay - bx
bz-cy
a
b
C
abz - acy
bcx - baz
a2
2
b2
cay – cbx c2
||
—
abz - acy + bcx - baz + cay - cbx
a2 + b2+c
2
Suy ra
bz-cy=0bz = cy⇒
C
y
b
x_y
=
a b
ay - bx = 0⇒ay = bx⇒
X
Suy ra
y_z
N|0
=
X y x
4 6 z
⇒x= 4k, y = 6k, z = 9k.
3
Mặt khác x + y + z = –1009.
Suy ra (4k)3 +(6k)3 + (9k)3
- 64k3 +216k3 +729k3 = 1009k3=-1009
=
= 0.
3
⇒ k3 = −1 ⇒ k = -1
Từ đó ta có x = -1.4 = -4;
y = -1.6=-6; z = −1.9 = −9.
Vậy ba số cần tìm là –4; –6; –9.
Bài toán 8. Cho các số thực a;
bc
=
=
ab
a+b b+c ста
b; c khác 0
са
Tính giá trị
(các mẫu
thỏa mãn
của biểu thức P =
3
a3 +b3 +c3
ab2 + bc2 + ca2
са
3 3
—=
-
a b C
Bài toán 6. Tìm các số thực x; y; z thỏa mãn (x+y): (5-z): (y + z): (9+ y) = 3:1:2:5.
Lời giải. Từ giả thiết và áp dụng tính chất dãy tỉ số bằng nhau ta có
thức khác 0).
Lời giải. Vì a, b, c = 0 nên từ giả thiết ta có
53
a+b
b+c
ab
bc
1 1
1
||
c+a
1 1
са
+ =
110
+
1
-10
||
+
-10
bacbac
b
5
P
=
3 a3 +b3 +c2
=
a b C
⇒ a = b = c.
Thay vào P ta được
3 3
a3 +b3 +c3
3
3
a
Suy ra → +
= 1.
b d
3
Từ b : d : 9 = 5 : 1 : 2, suy ra
với t e Z, t#0
Do đó b = 5t, d = t, g = 2t.
C e +
—
g
=
3
-3
70
3k 4k
Bài toán 9. Tìm số tự nhiên có ba chữ số biết Thay vào (1) ta được rằng số đó chia hết cho 18 và các chữ số của nó tỉ lệ với 1; 2; 3.
+ +
5t t
k 71
-213
k -3
t 10
70
t 7
a -9
-12 e
-15
Do đó
=
=
5
"
b 35 d 7 g
14
-9
"
35
Lời giải. Gọi ba chữ số của số cần tìm là a, b, c.
Theo bài ra ta có a : b :c=1:2:3.
Từ đó suy ra
a b
1 2 3
Vì số đó chia hết cho 18 nên tổng các chữ số của số đó chia hết cho 2 và chia hết cho 9. Suy ra a + b +c: 9.
Áp dụng tính chất dãy tỉ số bằng nhau ta có
a b
1
2
=
C
= k⇒ a = k; b = 2k; c = 3k.
——
3
Vì c = 3k < 9 nên k <3.
Suy ra a + b + c = 6k : 9.
a c
Từ đó k : 3 mà k khác 0 nên k = 3.
Do đó a = 3, b = 6, c = 9.
Vì số đó chia hết cho 2 nên chữ số hàng đơn vị là 6.
Vậy số cần tìm là 396 hoặc 936.
Bài toán 10. Tìm ba phân số có tổng bằng
3
-3 Biết tử của chúng tỉ lệ với 3; 4; 5 còn
70
mẫu của chúng tỉ lệ với 5; 1; 2.
Lời giải. Gọi ba phân số cần tìm là
(với a, b, c, d, e, g là số nguyên và khác 0).
Vậy ba phân số cần tìm là
||
d g
==
1
=
8|2
=
5k -213
=
2t 70
-12
"
-15
7 14
t
Bài toán 11. Độ dài ba cạnh của một tam giác tỉ lệ với 2; 3; 4. Hỏi độ dài ba đường cao của tam giác tỉ lệ với ba số nào?
Lời giải. Gọi a, b, c là độ dài ba cạnh của một tam giác và ha,hp,hc lần lượt là các chiều cao tương ứng.
Ta có SABC
=
aha bhp chc
2
2
2
Suy ra aha = bh = ch . (1)
C'
Vì a, b, c tỉ lệ với 2, 3, 4 nên
a
2
b
= — =
C
=
k (k > 0).
3 4
Từ đó a = 2k, b = 3k, c = 4k.
Từ (1) suy ra 2k.ha = 3k.h = 4k.h
2ha 3hp 4hc
=
hahb
=
=
12 12 12 6 4 3
Vậy độ dài ba đường cao của tam giác tỉ lệ với 6; 4: 3.
"
; Bài tập vận dụng
Bài 1. Cho các số thực a, b, c, d thỏa mãn
ace
b'd g
b, d, g
a
C
=
bd:
Theo giả thiết ta có a : c : e=3 : 4 : 5; b : d :
a C e
g = 5 : 1 : 2 và
—
—
—
+ + b d g
:
=
3
-3
(1)
70
a
=
3
Từ a : c : e = 3 : 4 : 5, suy ra
với ke Z.
Suy ra a = 3k, c = 4k, e = 5k.
014
e
=
015
= k
54
b d
Chứng minh rằng
7a2 + 3ab 11a2 - 8b2
7c2+3cd
11c2 - 8d2
(với các mẫu khác 0).
Bài 2. Lớp 7A có 52 học sinh được chia làm ba tổ. Nếu tổ 1 bớt đi 1 học sinh, tổ 2 bớt đi 2 học sinh, tổ 3 thêm vào 3 học sinh thì số học sinh tổ một, tổ hai, tổ ba tỉ lệ nghịch với 3; 4; 2. Tìm số học sinh mỗi tổ.
THÁCH ĐẤU
Kết quả
TRẬN ĐẤU THỨ MỘT TRĂM NĂM MƯƠI LĂM
Người thách đấu: Võ Quốc Bá Cẩn, trường THCS Archimedes Academy, Hà Nội. Bài toán thách đấu: Cho các số thực dương x, y thỏa mãn (x + 1)(y + 1) = 4xy.
Chứng minh rằng:
1
1
+
≤1.
√3x2+1 √√3y2+1
Thời hạn: Trước ngày 08.10.2018 theo dấu bưu điện.
TRẬN ĐẤU THỨ MỘT TRĂM NĂM MƯƠI HAI (TTT2 số 180+181)
Đề bài. Tìm nghiệm nguyên không âm của phương trình x(8x + 12x + 6) = y(y + y +1). Lời giải. Phương trình đã cho tương đương với (2x+1)3 = (y + 1)(y2 +1). (1)
4
Vì (2x+1)3 lẻ nên (y+1), (y^ + 1) lẻ.
a) Xét y = 0 thì a = b = 1; X = 0 (thỏa mãn đề bài). b) Xét số chẵn y22.
4
Gọi d = (y + 1; y“ +1) thì d là số lẻ.
Vì y4 −1= (y2 + 1)(y - 1)(y + 1) nên
4
4
d = (y + 1; y2 + 1) = (y + 1; y* − 1 + 2) = (y + 1; 2). Mà d lẻ nên d = 1.
―
=
* Nếu b = 6k 1 thì y b3 không chia hết cho 6 (loại).
Do b≥ 3, b = 6k + 1 > 7.
3
―
1
―
= (6k 1)3 1
―
Từ (2) có a = y^ + 1 = (b − 1)4 +1 =b12-4b9 + 6b6 - 4b3 + 2. (4)
Từ (4) dễ thấy số (b^ - 2b)3 = bở(b® – 2)3 = b12 – 6b9 + 12b^ – 8b3 < a với b 2 7.
Do đó chỉ có thể xảy ra:
3
3
a = (b^ − b − n) với số nguyên n mà 1≤n 17; b ≥ 3 và (a, b) = 1. Từ (2) ta có
3
3
y = (b3 - 1)4 = a3 − 1 = (a − 1)(a2 + a + 1). (3)
2
-
Gọi c = (a−1, a +a+1); (ce N*).
2
Vì a +a+1=(a−1)(a+2)+3 nên
2
c = (a−1; a + a + 1) = (a − 1; 3) ⇒ c ɛ {1; 3}.
⚫ TH1. c = 1.
Từ (3) có (a−1)(a +a+1) = (b−1)4 là số chính
phương.
Vì (a−1; a + a + 1) = 1 và a−1; a +a+len*
2
nên a−1; a+a+1 là các số chính phương.
2
Đặt a +a+1=k? (k = N*) thì
(2k)2 − (2a + 1)2 = 3
—
⇒ (2k-2a − 1)(2k + 2a + 1) = 3.
Mà k, ae N*, a>3, suy ra phương trình không có
nghiệm nguyên dương.
• TH2. c = 3 là ước của a – 1.
3
Từ (2) có y^ = a – 1 nên y : 3.
Do y chẵn nên b3 – 1 = y : 6.
y
y:
Hơn nữa y + 1 = b là số lẻ và b không chia hết cho 3 nên b = 6k + 1 hoặc b = 6k – 1.
4 1
27
16 4
+ b +
3 3
9
+-b+ ;(44b3 –53) > 0 với b 2 7.
+10+ (4403
* Khi n = 2k+1 =
từ (5) ta có
b+2
"
3
e-a3
3
==
-b° 3
3
3
44
+
-b3
3
62
27
9
9
27
2
2
=
3
32
9
32b2-16b
·b1(3b4 − 8b + 2) ·
b2
16 62 --b-
—
9 17
Như vậy với b ≥7 thì
63
-64
(
22
3b3
3
< 0 với b ≥ 7
(b+ - b - 2k − 1)3 < a3 < (b4 − b − 2k)3
b4-b - 2k - 1 < a < b4 - b - 2k.
Suy ra không tồn tại số nguyên a với b 2 7.
Tóm lại phương trình ban đầu có nghiệm nguyên không âm duy nhất (x; y) = (0; 0).
Nhận xét. Đây là bài toán khó nên không có võ sĩ nào giải trọn vẹn.
NGUYỄN VIỆT HẢI
55
DÀNH CHO CÁC NHÀ TOÁN HỌC NHỎ
MỞ RỘNG VÀ KHAI THÁC MỘT SỐ BÀI TOÁN TỔ HỢP
ThS. TRỊNH HOÀI DƯƠNG (GV. THCS Giảng Võ, Ba Đình, Hà Nội) ThS. LÊ ĐÌNH TRƯỜNG (Học viện Aladdin)
(Tiếp theo kì trước)
Bài toán 2. Cho X là một tập hợp gồm 700 số nguyên dương đôi một khác nhau, mỗi số không lớn hơn 2006. Chứng minh trong tập hợp X luôn tìm được hai phần tử x, y sao cho x − y thuộc tập hợp E = {3; 6; 9}.
(Đề thi vào lớp 10 THPT chuyên Đại học Sư phạm Hà Nội năm học 2006 - 2007) Lời giải. Cách 1. Theo nguyên lí Dirichlet thì trong 700 = 3.233 + 1 số có ít nhất 234 số có cùng số dư khi chia cho 3. Gọi 234 số đó là a1, a2, a234 với 1 ≤ a < a < ... < 8234 ≤ 2006.
Giả sử không tồn tại hai số a, a nào thỏa mãn a – aj = {3; 6; 9}. Do đó a
―
a ≥ 12 (vì
a – a : 3 và a + a). Trong 234 số trên, hai
aj
số kề nhau hơn kém nhau ít nhất 12 đơn vị nên a234 – a ≥ 234.12 = 2808 > 2006 (vô lí). Vậy điều giả sử là sai, suy ra đpcm. Chúng ta đi khai thác bài toán trên
* Hướng khai thác 1. Tìm thêm các cách giải cho bài toán trên.
Cách 2. Xét 700 số nguyên dương đã cho đôi một khác nhau và sắp thứ tự như sau:
1 ≤ a1 7 (mẫu thuẫn với (1)).
>
Vậy a, az, a là ba số đôi một nguyên tố cùng nhau. Bổ đề được chứng minh.
K
• Quay trở lại bài toán: Ta chia tập hợp A thành 6 tập hợp, mỗi tập hợp gồm 6 số tự nhiên liên tiếp A = {6k + 1; 6k + 2; ... ; 6k + 6} với k 0, 1, 2, 3, 4, 5. Theo nguyên lí Dirichlet thì trong 25 phần tử của tập hợp A thì có ít nhất 5 phần tử thuộc cùng một tập hợp A. Theo bổ đề trên ta có đpcm.
=
* Hướng khai thác 2. Tổng quát bài toán: Bản chất bài toán là chỉ ra 5 số cùng thuộc tập hợp A = {6k + 1; 6k + 2; ... ; 6k + 6} với k
; = 0, 1, 2, 3, 4, 5. Tức là cho 36 số nguyên dương từ 1 đến 36 thì ta cần lấy 25 = 6.4 + 1 số khác nhau trong 36 số đã cho.
Ta cho k nhận các giá trị khác, chẳng hạn từ 0 đến 499 tức là cho 3000 số nguyên dương từ 1 đến 3000 thì ta cần lấy 2001 = 500.4 + 1 số khác nhau trong 3000 số đã cho thì luôn tìm được 3 số đôi một nguyên tố cùng nhau. Từ đó ta có bài toán tổng quát:
Bài toán 3.1. Cho tập hợp A gồm 6k số tự nhiên liên tiếp từ 1 đến 6k (k ∈ N, k > 1). Chứng minh rằng trong 4k + 1 phần tử bất kì của tập hợp A luôn tìm được 3 phần tử là 3 số đôi một nguyên tố cùng nhau.
lời giải thứ hai.
* Hướng khai thác 3. Các bài toán tương tự. Bài toán 3.2. Cho tập hợp A gồm 36 số tự nhiên liên tiếp từ 1 đến 36. Tìm số tự nhiên n nhỏ nhất sao cho mọi tập hợp con n phần tử của A đều chứa 3 phần tử là các số đôi một nguyên tố cùng nhau.
Lời giải. Trước hết ta chứng minh n >24. Đặt M, = {2; 4; 6; ... ; 36}; M3 = {3; 6; 9;
58
36}; M1 = M2 ~ M3; M4 = M2 ~ M3. Ta thấy tập hợp M, có 18 phần tử, tập hợp Mỹ có 12 phần tử, tập hợp M, có 6 phần tử. Do đó tập hợp M, có 18 + 12 – 6 = 24 phần tử.
Theo nguyên lí Dirichlet, mọi tập hợp con của M, chứa ít nhất 3 phần tử đều có 2 phần tử nằm trong M, hoặc Mỹ, hai phần tử này không nguyên tố cùng nhau.
;
Do đó n > 24. Ta chứng minh tiếp n = 25 thỏa mãn yêu cầu của đề bài như Bài toán 3. Vậy n nhỏ nhất là 25.
Một cách tự nhiên ta có bài toán tổng quát: Bài toán 3.3. Cho tập hợp A gồm 6k số tự nhiên liên tiếp từ 1 đến 6k (k = N, k > 1). Tìm
số tự nhiên n nhỏ nhất sao cho mọi tập hợp con n phần tử của A đều chứa 3 phần tử là các số đôi một nguyên tố cùng nhau.
Hướng dẫn. Với lập luận tương tự bài toán 1
ta chứng minh được 4k + 1 là số tự nhiên nhỏ nhất thỏa mãn yêu cầu của bài toán.
Như vậy trong trường hợp tập hợp A gồm 6k phần tử và số các phần tử nguyên tố cùng nhau là 3 thì bài toán được giải quyết. Bây
3 giờ nếu ta thay đổi các dữ kiện của bài toán thì sao? Các bạn hãy giải bài toán IMO 1991 sau đây:
Bài toán 3.4. Cho tập hợp A gồm 280 số tự nhiên liên tiếp từ 1 đến 280. Tìm số tự nhiên n nhỏ nhất sao cho mọi tập hợp con n phần tử của A đều chứa 5 phần tử là các số đôi một nguyên tố cùng nhau.
Bài toán 4. Có bao nhiêu số nguyên dương có ba chữ số abc thỏa mãn a≤b≤c?
(IMSO 2015 tại Thái Lan)
Lời giải. • Nếu a = b = c thì có 9 số.
Nếu a = b hoặc b = c thì có
2.C3.1=2x
9×8
2x1
=72 số.
9!
=84 số.
3!(9-3)!
• Nếu a
1
(Vi=1,n−1). (*)
a¡ ai+1 30 Ta sẽ chứng minh n ≤ 10.
Thật vậy giả sử ngược lại n ≥ 11. (1)
1 1
1
Từ (*) ta có
> (Vi = 6,n−1).
a¡
ai+1
30
1
1
n-6
Suy ra E
a¡
ai+1
30
1
1
n-6
ΔΙ
аб
an
30
1
1 n-6 1 > >
+
n-6
(vì a6 >6)
(24; 32; 40); (9; 40; 41) và (27; 36; 45). Như vậy ta chỉ cần loại đi 4 phần tử 8, 9, 24, 36 thì tập hợp A có 41 phần tử còn lại thỏa mãn đề bài.
* Hướng khai thác 1. Phát biểu bài toán dưới dạng khác, dạng cực trị tổ hợp: Bài toán 5.1. Cho tập hợp
S={xe Z|1≤x≤50}.
Xét A là một tập hợp con bất kì của tập hợp S và A có n phần tử. Tìm giá trị lớn nhất của n để tập hợp A có tính chất: Không có ba phần tử nào của tập hợp A là số đo độ dài ba cạnh của một tam giác vuông.
60
6 аб 30 an 30 =n<11 (mâu thuẫn với (1)). Suy ra điều giả sử là sai. Do đó n < 10.
Mặt khác, ta chỉ ra được một tập hợp S có 10 phần tử là S = {1; 2; 3; 4; 5; 6; 8; 11; 18; 45}. Từ đó, tập hợp S có thể có nhiều nhất là 10 phần tử.
PROBLEM SOULING
ENGLISH
THOUGH
ENGLISH THROUGH PROBLEM SOLVING
GIẢI TOÁN
23
Key word
PERPENDICULAR LINES
AND PARALLEL LINES
TRỊNH HOÀI DƯƠNG (GV. THCS Giảng Võ, Ba Đình, Hà Nội) NGUYỄN THÀNH NAM (Cựu SV Đại học Oxford, Vương quốc Anh) HOÀNG ANH QUÂN (Cử nhân tài năng toán, Đại học KHTN Hà Nội)
Point, Segment, Line, Ray, Plane, Angle, Parallel, Perpendicular, Supplementary,
Complementary.
Transversal
Alternate angles
Alternate exterior angles Corresponding angles Definition
cát tuyến
các góc so le (trong)
các góc so le ngoài các góc đồng vị
When two lines cut to form right angles (90°), we say that the lines are perpendicular to each other.
- If M is the midpoint of AB, and PM is perpendicular to AB, PM is called the perpendicular bisector of AB.
- Two lines in a plane are parallel lines if they have not a common point.
are
- Two angles are said to be complementary if their sum is 90°. If two angles complementary, each angle is called the complement of the other.
- Two angles are said to be supplementary if their sum is 180°. If two angles are supplementary, each angle is called the supplement of the other.
- In Figure 1, two lines cut at G to form four angles. ZAGD and BGC are called vertically opposite angles, and so are ZBGD and ZAGC.
Properties
D
A
G
B
C
Figure 1
A line that is perpendicular to one of two parallel lines is also perpendicular to the other.
A line that is parallel to one of two parallel lines is also parallel to the other.
• When a transversal cuts two parallel lines: - The alternate angles are equal,
- The corresponding angles are equal,
- The interior angles on the same side of the transversal are supplementary.
Two lines in a plane are parallel if they are cut by a transversal in such a way that: - The alternate angles are equal, or
- The corresponding angles are equal, or
- The interior angles on the same side of the transversal are supplementary.
Exercise
In the figure below, lines g and f are parallel. Prove that po = m° +n°.
A
g
mo
B
X
f
Solution
y
n°
C
Draw a line xy such that line xy goes through point B and it is parallel to line g, as shown in the figure.
The transversal AB cuts two parallel lines g and xy, it impies that the alternate angles ZgAB and ZABy are equal. Similarly, two angles /fCB and CBy are equal. Therefore, ZABC = ZABY + ZCBy
=
2ab
b2 a2+4b2 3a2+2b2-5
+
ĐẬU CÔNG NHO (GV. THCS Cao Xuân Huy, Diễn Châu,
Nghệ An)
Bài 7(185+186). Một giải bóng đá gồm các đội trong nước và một số đội bóng nước ngoài với thể thức thi đấu vòng tròn một lượt. Số đội bóng trong nước gấp 5 lần số đội bóng nước ngoài. Kết thúc giải thì tổng số điểm của các đội trong nước gấp đôi tổng số điểm của các đội nước ngoài. Biết rằng không có trận nào hòa và ở mỗi trận đấu thì đội thắng được 3 điểm, đội thua được 0 điểm. Hỏi có bao nhiêu đội nước ngoài và cho biết thứ tự trong bảng xếp hạng của các đội nước ngoài?
VÕ XUÂN MINH (GV. THCS Nguyễn Văn Trỗi, Cam Nghĩa, Cam Ranh, Khánh Hòa)
Bài 8(185+186). Cho hình vuông ABCD với M, N lần lượt là trung điểm của BC và CD; AM cắt BN tại điểm O. Biết diện tích tứ giác AOND bằng 55 cm. Tính độ dài cạnh của hình vuông.
LÊ TRẦN QUỐC CẢNH
(GV. THCS Gia Lộc, Trảng Bàng, Tây Ninh)
SOLVE VIA MAIL COMPETITION QUESTIONS Translated by Trang Duong Thu
5(185+186). Find the positive integers n such that 3" + 4" + 2018" is a square
number.
6(185+186). Find whole numbers a, b such
3
that (a, b) = 1 and
2ab
+
b2
> a2+4b2 3a2+2b2-5
7(185+186). In a football match with local and foreign teams, each team plays one game with each other. The number of local teams is 5 times the number of foreign
teams. At the end, the total score of local teams is twice the total score of foreign teams. No game is draw. Win gets 3 score and loss gets 0 score. How many foreign teams are there? What are the foreign
teams' places in the ranking score table?
8(185+186). Let ABCD be a square with M, N as the midpoint of BC and CD respectively. AM intersects BN at O. The area of quadrilateral AOND is 55 cm2. Find the length of the square's side.
itphotos
64
LTS. TS. Lê Thống Nhất, một trong những người sáng lập “Toán Tuổi thơ” từ năm 2000. Sau 10 năm tạm biệt Toán Tuổi thơ để sáng lập các cuộc thi trên mạng như ViOlympic, IOE và Trường học lớn BigSchool, thầy đã trở về tham gia Hội đồng biên tập để cùng xây dựng, phát triển Tạp chí. Thầy đã tâm sự với bạn đọc qua bài thơ “Tâm sự ngày trở về”.
Tâm sự sự
2
ngày trở về...
chu đi rồi lại trở về
Bồi hồi nhớ thuở đam mê ngày nào
Mười năm... nhanh thế rồi sao?
Mn lừng chuyên mục, nghẹn ngào lòng là
Thầy bộ muộn nên gần
xa
học tên cả nước như là mẫu thân
Biết bao bài viết rất cầu
Biết bao câu hỏi xoay nền cùng nhau
long là học giỏi, biển mau Vừa chơi, vừa học mai sau thành tài
long Thầy bê mẫu miệt mùi
Súp thêm h luệ qua bài viết hay Bóp bí
ellong cho tạp chỉ đổi thay
ellong cho bạn đọc mỗi ngày thêm động
Cùng nhau gian khi nen hồng
vun
khi mong bi ngọt, còn công cá gì
ဘာ
LÊ THỐNG NHẤT
Dành cho giáo viên, phụ huynh và trẻ em từ 12 tuổi đến dưới 16 tuổi.
...
các m
Giấy phép xuất bản: số 31/GP-BVHTT, cấp ngày 23/1/2003 của Bộ Văn hóa và Thông tin. Mã số: 8BTT185M18. In tại: Công ty cổ phần in Công Đoàn Việt Nam, 167 Tây Sơn, Đống Đa, Hà Nội. In xong và nộp lưu chiểu tháng 9 năm 2018.
Giá: 20 000 đồng
"""